Ch. 26-33 aka hell set

¡Supera tus tareas y exámenes ahora con Quizwiz!

The client with insulin-dependent type 2 diabetes and an HbA1c of 5.0% is planning to become pregnant soon. What anticipatory guidance should the nurse provide this client? 1. Insulin needs decrease in the first trimester and usually begin to rise late in the first trimester as glucose use and glycogen storage by the woman and fetus increase. 2. The risk of ketoacidosis decreases during the length of the pregnancy. 3. Vascular disease that accompanies diabetes slows progression. 4. The baby is likely to have a congenital abnormality because of the diabetes.

1 Explanation: 1. Insulin needs decrease in the first trimester and usually begin to rise late in the first trimester as glucose use and glycogen storage by the woman and fetus increase.

A client with diabetes is receiving preconception counseling. The nurse will emphasize that during the first trimester, the woman should be prepared for which of the following? 1. The need for less insulin than she normally uses 2. Blood testing for anemia 3. Assessment for respiratory complications 4. Assessment for contagious conditions

1 Explanation: 1. Women with diabetes often require less insulin during the first trimester.

MATCHING Because most pregnant women continue their usual activities, trauma remains a common complication during pregnancy. Approximately 1 in 12 pregnancies in the United States are complicated by trauma each year. As a result of the physiologic alterations that accompany pregnancy, special considerations for the mother and fetus are necessary when trauma occurs. Match the maternal system adaptation in pregnancy with the clinical response to trauma. a. Increased oxygen consumption b. Increased heart rate c. Decreased gastric motility d. Displacement of abdominal viscera e. Increase in clotting factors 1. Decreased placental perfusion in the supine position 2. Increased risk of thrombus formation 3. Altered pain referral 4. Increased risk of acidosis 5. Increased risk of aspiration

1. ANS: B DIF: Cognitive Level: Apply REF: p. 732 TOP: Nursing Process: Assessment MSC: Client Needs: Physiologic Integrity NOT: Immediate priorities for the stabilization of the pregnant woman after trauma should be identical to that of the nonpregnant client after trauma. Fetal survival depends on maternal survival, and stabilization of the mother improves the chance of fetal well-being. Trauma may affect a number of systems within the body, and being aware of normal system alterations in the pregnant woman is important for the nurse who is caring for this client. Care should be adapted according to the body system that has been injured. The effects of trauma on pregnancy are also influenced by the length of gestation, type and severity of the injuries, and the degree of disruption of uterine and fetal physiologic features. 2. ANS: E DIF: Cognitive Level: Apply REF: p. 732 TOP: Nursing Process: Assessment MSC: Client Needs: Physiologic Integrity NOT: Immediate priorities for the stabilization of the pregnant woman after trauma should be identical to that of the nonpregnant client after trauma. Fetal survival depends on maternal survival, and stabilization of the mother improves the chance of fetal well-being. Trauma may affect a number of systems within the body, and being aware of normal system alterations in the pregnant woman is important for the nurse who is caring for this client. Care should be adapted according to the body system that has been injured. The effects of trauma on pregnancy are also influenced by the length of gestation, type and severity of the injuries, and the degree of disruption of uterine and fetal physiologic features. 3. ANS: D DIF: Cognitive Level: Apply REF: p. 732 TOP: Nursing Process: Assessment MSC: Client Needs: Physiologic Integrity NOT: Immediate priorities for the stabilization of the pregnant woman after trauma should be identical to that of the nonpregnant client after trauma. Fetal survival depends on maternal survival, and stabilization of the mother improves the chance of fetal well-being. Trauma may affect a number of systems within the body, and being aware of normal system alterations in the pregnant woman is important for the nurse who is caring for this client. Care should be adapted according to the body system that has been injured. The effects of trauma on pregnancy are also influenced by the length of gestation, type and severity of the injuries, and the degree of disruption of uterine and fetal physiologic features. 4. ANS: A DIF: Cognitive Level: Apply REF: p. 732 TOP: Nursing Process: Assessment MSC: Client Needs: Physiologic Integrity NOT: Immediate priorities for the stabilization of the pregnant woman after trauma should be identical to that of the nonpregnant client after trauma. Fetal survival depends on maternal survival, and stabilization of the mother improves the chance of fetal well-being. Trauma may affect a number of systems within the body, and being aware of normal system alterations in the pregnant woman is important for the nurse who is caring for this client. Care should be adapted according to the body system that has been injured. The effects of trauma on pregnancy are also influenced by the length of gestation, type and severity of the injuries, and the degree of disruption of uterine and fetal physiologic features. 5. ANS: C DIF: Cognitive Level: Apply REF: p. 732 TOP: Nursing Process: Assessment MSC: Client Needs: Physiologic Integrity NOT: Immediate priorities for the stabilization of the pregnant woman after trauma should be identical to that of the nonpregnant client after trauma. Fetal survival depends on maternal survival, and stabilization of the mother improves the chance of fetal well-being. Trauma may affect a number of systems within the body, and being aware of normal system alterations in the pregnant woman is important for the nurse who is caring for this client. Care should be adapted according to the body system that has been injured. The effects of trauma on pregnancy are also influenced by the length of gestation, type and severity of the injuries, and the degree of disruption of uterine and fetal physiologic features.

Nursing intervention for a pt with eclampsia after a siezure

1. Assess status of the woman's airway, breathing and pulse 2. Suction secretions from glottis to clear airway, 3. administer O2 4. Start IV if not in place or if previous infiltrated 5. Administer mg sulfate therapy

What assessment should be performed to assess for hypertensive disorders?

1. BP 2. Edema 3. DTRs (refelcct balance bt cerebral cortex and spinal cord) 4. assess for ankle clonus 5. Presence of proteinuria 6. Evaluate for s & s of severe preeclampsia: headache, epigastric pain, RUQ, visual disturbances

Nursing interventions for severe gestational hypertension and preeclampsia with severe features

1. Bed rest with side rails up 2. darkened enviroment 3. Mg sulfate therapy 4. anti hypertensive medications

Nursing intervention for a pt with eclampsia during seizure

1. Priority care: keep airway patent: turn head to one side, place pillow under one shoulder if possible 2. Call for assistance. Do not leave bedside 3. Raise side rails and pad them with a folded blanket or pillow if possible 4. Observe and record convulsion activity (note time and duration)

What are the systemic effects as a result of poor tissue perfusion and reduced plasma volume in a pt with preeclampsia?

1. Reduced kidney perfusion= decrease in glomerular filtration rate= protein (albumin) lost in urine, retaining of Na+ and H2O; necrosis and renal failure can occur 2. Decrease in plasma colloid osmotic pressure from decrease in levels of serum albumin= decrease in intravascular vol= hemoconcentration, increased blood viscosity, and tissue edema; increase in hct value = arteriolar vasospasm = endothelial damage and increased cap permeability= pulmonary edema 3. Decreased liver perfusion= impaired liver fxn and elevated liver enzyme levels; n&v, epigastric pain, RUQ pain 4. Cerebral edema and hemorrhage in increased CNS irritability (headaches, hyperflexia, clonus in ankle, seizures); arteriolar vasospasms and decreased blood flow to retina= visual disturbances

What is intrapartum nursing care directed toward for a pt diagnosed with severe gestational htn or preeclampsia with severe features?

1. early identification of fetal heart rate abnormalities 2. Prevention of maternal complications

What is HELP syndrome characterized by?

1. hemolysis 2. elevated liver enzymes 3. low platelet

Name four common risk factors for preeclampsia

1. primigravidity in woman <19 or >40 2. First pregnancy with new partner 3. Hx of preeclampsia 4. pregnancy-onset snoring

A newly diagnosed insulin-dependent type 1 diabetic with good blood sugar control is at 20 weeks gestation. She asks the nurse how her diabetes will affect her baby. What would the best explanation include? 1. Your baby could be smaller than average at birth. 2. Your baby will probably be larger than average at birth. 3. As long as you control your blood sugar, your baby will not be affected at all. 4. Your baby might have high blood sugar for several days.

2 Explanation: 2. Characteristically, infants of mothers with diabetes are large for gestational age (LGA) as a result of high levels of fetal insulin production stimulated by the high levels of glucose crossing the placenta from the mother. Sustained fetal hyperinsulinism and hyperglycemia ultimately lead to excessive growth, called macrosomia, and deposition of fat.

A 26-year-old client is 28 weeks pregnant. She has developed gestational diabetes. She is following a program of regular exercise, which includes walking, bicycling, and swimming. What instructions should be included in a teaching plan for this client? 1. Exercise either just before meals or wait until 2 hours after a meal. 2. Carry hard candy (or other simple sugar) when exercising. 3. If your blood sugar is 120 mg/dL, eat 20 g of carbohydrate. 4. If your blood sugar is more than 120 mg/dL, drink a glass of whole milk.

2 Explanation: 2. The nurse should advise her to carry a simple sugar such as hard candy because of the possibility of exercise-induced hypoglycemia.

A 26-year-old client is 26 weeks pregnant. Her previous births include two large-for-gestational-age babies and one unexplained stillbirth. Which tests would the nurse anticipate as being most definitive in diagnosing gestational diabetes? 1. A 50g, 1-hour glucose screening test 2. A single fasting glucose level 3. A 100g, 1-hour glucose tolerance test 4. A 100g, 3-hour glucose tolerance test

4 Explanation: 4. Gestational diabetes is diagnosed if two or more of the following values are met or exceeded after taking the 100 g, 3-hour OGTT: Fasting: 95 mg/dL; 1 hour: 180 mg/dL; 2 hours: 155 mg/dL; 3 hours: 140 mg/dL.

The client has just been diagnosed as diabetic. The nurse knows teaching was effective when the client makes which statement? 1. Ketones in my urine mean that my body is using the glucose appropriately. 2. I should be urinating frequently and in large amounts to get rid of the extra sugar. 3. My pancreas is making enough insulin, but my body isnt using it correctly. 4. I might be hungry frequently because the sugar isnt getting into the tissues the way it should.

4 Explanation: 4. The client who understands the disease process is aware that if the body is not getting the glucose it needs, the message of hunger will be sent to the brain.

2. Which condition would require prophylaxis to prevent subacute bacterial endocarditis (SBE) both antepartum and intrapartum? a. Valvular heart disease b. Congestive heart disease c. Arrhythmias d. Postmyocardial infarction

: A Prophylaxis for intrapartum endocarditis and pulmonary infection may be provided for women who have mitral valve prolapse. Prophylaxis for intrapartum endocarditis is not indicated for a client with congestive heart disease, underlying arrhythmias, or postmyocardial infarction. DIF: Cognitive Level: Understand REF: p. 712 TOP: Nursing Process: Implementation

8. Since the gene for cystic fibrosis was identified in 1989, data can be collected for the purposes of genetic counseling for couples regarding carrier status. According to the most recent statistics, how often does cystic fibrosis occur in Caucasian live births? a. 1 in 100 b. 1 in 1000 c. 1 in 2000 d. 1 in 3200

: D Cystic fibrosis occurs in approximately 1 in 3200 Caucasian live births. 1 in 100, 1 in 1000, and 1 in 2000 occurrences of cystic fibrosis in live births are all too frequent rates. DIF: Cognitive Level: Remember REF: p. 722 TOP: Nursing Process: Assessment

Screening questions for alcohol and drug abuse should be included in the overall assessment during the first prenatal visit for all women. The 4 Ps-Plus is a screening tool designed specifically to identify when there is a need for a more in-depth assessment. Which of the following is not included in the 4 Ps-Plus screening tool? a. Present b. Partner c. Past d. Pregnancy

A

26. Intrauterine growth restriction (IUGR) is associated with numerous pregnancy-related risk factors (Select all that apply). a. Poor nutrition b. Maternal collagen disease c. Gestational hypertension d. Premature rupture of membranes e. Smoking

A, B, C, E

Medications used to manage postpartum hemorrhage (PPH) include (Select all that apply): a. Pitocin. b. Methergine. c.Terbutaline. d. Hemabate. e. Magnesium sulfate.

A, B, D Pitocin, Methergine, and Hemabate are all used to manage PPH. Terbutaline and magnesium sulfate are tocolytics; relaxation of the uterus causes or worsens PPH.

2. Screening questions for alcohol and drug abuse should be included in the overall assessment during the first prenatal visit for all women. The 4 Ps Plus is a screening tool specifically designed to identify the need for a more in-depth assessment. Which are the correct components of the 4 Ps Plus? (Select all that apply.) a. Parents b. Partner c. Present d. Past e. Pregnancy

ABDE

MULTIPLE RESPONSE 1. A client who has undergone a D&C for early pregnancy loss is likely to be discharged the same day. The nurse must ensure that her vital signs are stable, that bleeding has been controlled, and that the woman has adequately recovered from the administration of anesthesia. To promote an optimal recovery, what information should discharge teaching include? (Select all that apply.) a. Iron supplementation b. Resumption of intercourse at 6 weeks postprocedure c. Referral to a support group, if necessary d. Expectation of heavy bleeding for at least 2 weeks e. Emphasizing the need for rest

ANS: A, C, E The woman should be advised to consume a diet high in iron and protein. For many women, iron supplementation also is necessary. The nurse should acknowledge that the client has experienced a loss, however early. She can be taught to expect mood swings and possibly depression. Referral to a support group, clergy, or professional counseling may be necessary. Discharge teaching should emphasize the need for rest. Nothing should be placed in the vagina for 2 weeks after the procedure, including tampons and vaginal intercourse. The purpose of this recommendation is to prevent infection. Should infection occur, antibiotics may be prescribed. The client should expect a scant, dark discharge for 1 to 2 weeks. Should heavy, profuse, or bright bleeding occur, she should be instructed to contact her health care provider. DIF: Cognitive Level: Apply REF: p. 672 TOP: Nursing Process: Implementation MSC: Client Needs: Physiologic Integrity

24. What is a maternal indication for the use of vacuum-assisted birth? a. Wide pelvic outlet b. Maternal exhaustion c. History of rapid deliveries d. Failure to progress past station 0

ANS: B A mother who is exhausted may be unable to assist with the expulsion of the fetus. The client with a wide pelvic outlet will likely not require vacuum extraction. With a rapid delivery, vacuum extraction is not necessary. A station of 0 is too high for a vacuum-assisted birth. DIF: Cognitive Level: Understand REF: p. 786 TOP: Nursing Process: Assessment MSC: Client Needs: Physiologic Integrity

16. The nurse has evaluated a client with preeclampsia by assessing DTRs. The result is a grade of 3+. Which DTR response most accurately describes this score? a. Sluggish or diminished b. Brisk, hyperactive, with intermittent or transient clonus c. Active or expected response d. More brisk than expected, slightly hyperactive

ANS: D DTRs reflect the balance between the cerebral cortex and the spinal cord. They are evaluated at baseline and to detect changes. A slightly hyperactive and brisk response indicates a grade 3+ response. DIF: Cognitive Level: Apply REF: p. 660 TOP: Nursing Process: Assessment MSC: Client Needs: Physiologic Integrity

What it essential for early detection of htn disorders in a pregnant woman?

Accurate measure of BP

3. During an inpatient psychiatric hospitalization, what is the most important nursing intervention? a. Contacting the client's significant other b. Supervising and guiding visits with her infant c. Allowing no contact with anyone who annoys her d. Having the infant with the mother at all times

B In the hospital setting, the reintroduction of the infant to the mother can and should occur at the mother's own pace. A schedule is set that increases the number of hours the mother cares for her infant over several days, culminating in the infant staying overnight in the mother's room. These supervised and guided visits allow the mother to experience meeting the infant's needs and giving up sleep for the infant. Reintroducing the mother to her infant while in a supervised setting is essential. Another important task for a mother under psychiatric care is to reestablish positive interactions with others.

The nurse would conclude that grieving parents had progressed to the reorganization/recovery phase during a follow-up visit a year later if: a. The parents say they feel no pain. b. The parents are discussing sex and a future pregnancy, even if they have not sorted out their feelings yet. c. The parents have abandoned those moments of bittersweet grief. d. The parents' questions have progressed from "Why?" to "Why us?"

B. The parents are discussing sex and a future Many couples have conflicting feelings about sexuality and future pregnancies. A little pain is always present, certainly past the first year, when recovery begins to peak. Bittersweet grief describes the brief grief response that occurs with reminders of a loss, often on anniversary dates. Most couples never abandon it. Recovery is ongoing. Typically a couple's search for meaning progresses from "Why?" in the acute phase to "Why me?" in the intense phase to "What does this loss mean to my life?" in the reorganizational phase.

During pregnancy, alcohol withdrawal may be treated using: a. Disulfiram (Antabuse). b. Corticosteroids. c. Benzodiazepines. d. Aminophylline.

C

7. Which is the most accurate description of PPD without psychotic features? a. Postpartum baby blues requiring the woman to visit with a counselor or psychologist b. Condition that is more common among older Caucasian women because they have higher expectations c. Distinguishable by pervasive sadness along with mood swings d. Condition that disappears without outside help

C PPD is characterized by an intense pervasive sadness along with labile mood swings and is more persistent than postpartum baby blues. PPD, even without psychotic features, is more serious and persistent than postpartum baby blues. PPD is more common among younger mothers and African-American mothers. Most women need professional help to get through PPD, including pharmacologic intervention.

8. While providing care to the maternity client, the nurse should be aware that one of these anxiety disorders is likely to be triggered by the process of labor and birth. Which disorder fits this criterion? a. Phobias b. Panic disorder c. Posttraumatic stress disorder (PTSD) d. Obsessive-compulsive disorder (OCD)

C PTSD can occur as the result of a past trauma such as rape. Symptoms of PTSD include re-experiencing the event, numbing, irritability, angry outbursts, and exaggerated startle reflex. With the increased bodily touch and vaginal examinations that occur during labor, the client may have memories of the original trauma. The process of giving birth may result in her feeling out of control. The nurse should verbalize an understanding and reassure the client as necessary. Phobias are irrational fears that may lead a person to avoid certain events or situations. Panic disorders may occur in as many as 3% to 5% of women in the postpartum period and are described as episodes of intense apprehension, fear, and terror. Symptoms of a panic disorder may include palpitations, chest pain, choking, or smothering. OCD symptoms include recurrent, persistent, and intrusive thoughts. The mother may repeatedly check and recheck her infant once he or she is born, although she realizes that this behavior is irrational. OCD is optimally treated with medications.

9. Which substance used during pregnancy causes vasoconstriction and decreased placental perfusion, resulting in maternal and neonatal complications? a. Alcohol b. Caffeine c. Tobacco d. Chocolate

C Smoking in pregnancy is known to cause a decrease in placental perfusion and is the cause of low-birth-weight infants. Prenatal alcohol exposure is the single greatest preventable cause of mental retardation. Alcohol use during pregnancy can cause high blood pressure, miscarriage, premature birth, stillbirth, and anemia. Caffeine may interfere with certain medications and worsen arrhythmias. Chocolate, particularly dark chocolate, contains caffeine that may interfere with certain medications.

When helping the mother, father, and other family members actualize the loss of the infant, nurses should: a. Use the words lost or gone rather than dead or died. b. Make sure that the family understands that it is important to name the baby. c. If the parents choose to visit with the baby, apply powder and lotion to the baby and wrap the infant in a pretty blanket. d. Set a firm time for ending the visit with the baby so the parents know when to let go.

C. If the parents choose to visit with the baby, apply powder, and lotion to the baby and wrap the infant in al pretty infant. Presenting the baby in a nice way stimulates the parents' senses and provides pleasant memories of their baby. Nurses must use dead and died to assist the bereaved in accepting reality. Although naming the baby can be helpful, it is important not to create the sense that parents have to name the baby. In fact, some cultural taboos and religious rules prohibit the naming of an infant who has died. Parents need different time periods with their baby to say goodbye. Nurses need to be careful not to rush the process.

1. Reports have linked third trimester use of selective serotonin uptake inhibitors (SSRIs) with a constellation of neonatal signs. The nurse is about to perform an assessment on the infant of a mother with a history of a mood disorder. Which signs and symptoms in the neonate may be the result of maternal SSRI use? (Select all that apply.) a. Hypotonia b. Hyperglycemia c. Shivering d. Fever e. Irritability

CDE Neonatal signs of maternal SSRI use include continuous crying, irritability, jitteriness, shivering, fever, hypertonia, respiratory distress, feeding difficulty, hypoglycemia, and seizures. The onset of signs and symptoms ranges from several hours to several days after birth, but the signs generally resolve within 2 weeks.

Which of these medications would be classified as a Category X substance that is not to be used during pregnancy? a. Lorazepam (Ativan) b. Alprazolam (Xanax) c. Chlordiazepoxide (Librium) d. Temazepam (Restoril)

D

Which physiologic alteration of pregnancy most significantly affects glucose metabolism? a. Pancreatic function in the islets of Langerhans is affected by pregnancy. b. Pregnant women use glucose at a more rapid rate than nonpregnant women. c. Pregnant women significantly increase their dietary intake. d. Placental hormones are antagonistic to insulin, thus resulting in insulin resistance.

D Placental hormones, estrogen, progesterone, and human placental lactogen (HPL) create insulin resistance. Insulin is also broken down more quickly by the enzyme placental insulinase. Pancreatic functioning is not affected by pregnancy. The glucose requirements differ because of the growing fetus. The pregnant woman should increase her intake by 200 calories a day.

13. What is the most dangerous effect on the fetus of a mother who smokes cigarettes while pregnant? a. Genetic changes and anomalies b. Extensive CNS damage c. Fetal addiction to the substance inhaled d. Intrauterine growth restriction

D The major consequences of smoking tobacco during pregnancy are low-birth-weight infants, prematurity, and increased perinatal loss. Cigarettes will not normally cause genetic changes or extensive CNS damage. Addiction to tobacco is not a usual concern related to the neonate.

The perinatal nurse caring for the postpartum woman understands that late postpartum hemorrhage (PPH) is most likely caused by: a. Subinvolution of the placental site. b. Defective vascularity of the decidua. c. Cervical lacerations. d. Coagulation disorders.

a. Subinvolution of the placental site. Late PPH may be the result of subinvolution of the uterus, pelvic infection, or retained placental fragments. Late PPH is not typically a result of defective vascularity of the decidua, cervical lacerations, or coagulation disorders.

Despite popular belief, there is a rare type of hemophilia that affects women of childbearing age. von Willebrand disease is the most common of the hereditary bleeding disorders and can affect males and females alike. It results from a factor VIII deficiency and platelet dysfunction. Although factor VIII levels increase naturally during pregnancy, there is an increased risk for postpartum hemorrhage from birth until 4 weeks after delivery as levels of von Willebrand factor (vWf) and factor VIII decrease. The treatment that should be considered first for the client with von Willebrand disease who experiences a postpartum hemorrhage is: a. Cryoprecipitate. b.Factor VIII and vWf. c. Desmopressin d. Hemabate.

c. Desmopressin Desmopressin is the primary treatment of choice. This hormone can be administered orally, nasally, and intravenously. This medication promotes the release of factor VIII and vWf from storage. Cryoprecipitate may be used; however, because of the risk of possible donor viruses, other modalities are considered safer. Treatment with plasma products such as factor VIII and vWf are an acceptable option for this client. Because of the repeated exposure to donor blood products and possible viruses, this is not the initial treatment of choice. Although the administration of this prostaglandin is known to promote contraction of the uterus during postpartum hemorrhage, it is not effective for the client who presents with a bleeding disorder.

When a woman is diagnosed with postpartum depression (PPD) with psychotic features, one of the main concerns is that she may: a. Have outbursts of anger. b. Neglect her hygiene. c. Harm her infant. d. Lose interest in her husband.

c. Harm her infant. Thoughts of harm to one's self or the infant are among the most serious symptoms of PPD and require immediate assessment and intervention. Although outbursts of anger, hygiene neglect, and loss of interest in her husband are attributable to PPD, the major concern would be the potential to harm herself or her infant.

Complicated bereavement: a. Occurs when, in multiple births, one child dies, and the other or others live. b. Is a state in which the parents are ambivalent, as with an abortion. c. Is an extremely intense grief reaction that persists for a long time. d. Is felt by the family of adolescent mothers who lose their babies.

c. Is an extremely intense grief reaction that persists for a long time.

If nonsurgical treatment for late postpartum hemorrhage is ineffective, which surgical procedure is appropriate to correct the cause of this condition? a. Hysterectomy b. Laparoscopy c. Laparotomy d. D&C

d. D&C

The nurse caring for a family during a loss may notice that survival guilt sometimes is felt at the death of an infant by the child's: a. Siblings. b. Mother. c. Father. d. Grandparents.

d. Grandparents. Survival guilt sometimes is felt by grandparents, because they believe that the death upsets the natural order of things. They are angry that they are alive and their grandchild is not.

Define eclampsia

the onset of seizure activity or coma in a woman with preeclampsia who has no hx of preexisting patho resulting in seizure activity

The nurse has written the nursing diagnosis Injury, Risk for for a diabetic pregnant client. Interventions for this diagnosis include which of the following? Note: Credit will be given only if all correct and no incorrect choices are selected. Select all that apply. 1. Assessment of fetal heart tones 2. Perform oxytocin challenge test, if ordered 3. Refer the client to a diabetes support group 4. Assist with the biophysical profile assessment 5. Develop an appropriate teaching plan

1, 2, 4 Explanation: 1. Reassuring fetal heart rate variability and accelerations are interpreted as adequate placental oxygenation. 2. The nurse would perform oxytocin challenge test (OCT)/contraction stress test (CST) and non-stress tests as determined by physician. 4. The nurse assists the physician in performing a biophysical profile assessment.

A diabetic client goes into labor at 36 weeks gestation. Provided that tests for fetal lung maturity are successful, the nurse will anticipate which of the following interventions? Note: Credit will be given only if all correct and no incorrect choices are selected. Select all that apply. 1. Administration of tocolytic therapy 2. Beta-sympathomimetic administration 3. Allowance of labor to progress 4. Hourly blood glucose monitoring 5. Cesarean birth may be indicated if evidence of reassuring fetal status exists

3, 4 Explanation: 3. There will be no attempt to stop the labor, as this can compromise the mother and fetus. 4. To reduce incidence of congenital anomalies and other problems in the newborn, the woman should be euglycemic (have normal blood glucose) throughout the pregnancy.

MULTIPLE RESPONSE 1. Which congenital anomalies can occur as a result of the use of antiepileptic drugs (AEDs) in pregnancy? (Select all that apply.) a. Cleft lip b. Congenital heart disease c. Neural tube defects d. Gastroschisis e. Diaphragmatic hernia

: A, B, C Congenital anomalies that can occur with AEDs include cleft lip or palate, congenital heart disease, urogenital defects, and neural tube defects. Carbamazepine and valproate should be avoided if all possible; they may cause neural tube defects. Congenital anomalies of gastroschisis and diaphragmatic hernia are not associated with the use of AEDs. DIF: Cognitive Level: Understand REF: p. 725 TOP: Nursing Process: Planning

3. In caring for a pregnant woman with sickle cell anemia, the nurse must be aware of the signs and symptoms of a sickle cell crisis. What do these include? (Select all that apply.) a. Fever b. Endometritis c. Abdominal pain d. Joint pain e. Urinary tract infection (UTI)

: A, C, D Women with sickle cell anemia have recurrent attacks (crises) of fever and pain, most often in the abdomen, joints, and extremities. These attacks are attributed to vascular occlusion when red blood cells (RBCs) assume the characteristic sickled shape. Crises are usually triggered by dehydration, hypoxia, or acidosis. Women with the sickle cell trait are usually at a greater risk for postpartum endometritis (uterine wall infection); however, this development is not likely to occur during the pregnancy and is not a sign for the disorder. Although women with sickle cell anemia are at an increased risk for UTIs, these infections are not an indication of a sickle cell crisis. DIF: Cognitive Level: Understand REF: p. 721 TOP: Nursing Process: Assessment

2. A lupus flare-up during pregnancy or early postpartum occurs in 15% to 60% of women with this disorder. Which conditions associated with systemic lupus erythematosus (SLE) are maternal risks? (Select all that apply.) a. Miscarriage b. Intrauterine growth restriction (IUGR) c. Nephritis d. Preeclampsia e. Cesarean birth

: A, C, D, E Maternal risks associated with SLE include miscarriage, nephritis, preeclampsia, and cesarean birth. IUGR is a fetal risk related to SLE. Other fetal risks include stillbirth and prematurity. DIF: Cognitive Level: Understand REF: p. 727 TOP: Nursing Process: Assessment

3. Which information should the nurse take into consideration when planning care for a postpartum client with cardiac disease? a. The plan of care for a postpartum client is the same as the plan for any pregnant woman. b. The plan of care includes rest, stool softeners, and monitoring of the effect of activity. c. The plan of care includes frequent ambulating, alternating with active range-of-motion exercises. d. The plan of care includes limiting visits with the infant to once per day.

: B Bed rest may be ordered, with or without bathroom privileges. Bowel movements without stress or strain for the woman are promoted with stool softeners, diet, and fluids. Care of the woman with cardiac disease in the postpartum period is tailored to the woman's functional capacity. The woman will be on bed rest to conserve energy and to reduce the strain on the heart. Although the woman may need help caring for the infant, breastfeeding and infant visits are not contraindicated. DIF: Cognitive Level: Understand REF: pp. 718-719 TOP: Nursing Process: Planning

9. Which information regarding the care of antepartum women with cardiac conditions is mostimportant for the nurse to understand? a. Stress on the heart is greatest in the first trimester and the last 2 weeks before labor. b. Women with class II cardiac disease should avoid heavy exertion and any activity that causes even minor symptoms. c. Women with class III cardiac disease should get 8 to 10 hours of sleep every day and limit housework, shopping, and exercise. d. Women with class I cardiac disease need bed rest through most of the pregnancy and face the possibility of hospitalization near term.

: B Class II cardiac disease is symptomatic with ordinary activity. Women in this category need to avoid heavy exertion and limit regular activities as symptoms dictate. Stress is greatest between weeks 28 and 32 of gestation, when hemodynamic changes reach their maximum. Class III cardiac disease is symptomatic with less-than-ordinary activity. These women need bed rest most of the day and face the possibility of hospitalization near term. Class I cardiac disease is asymptomatic at normal levels of activity. These women can perform limited normal activities with discretion, although they still need a good amount of sleep. DIF: Cognitive Level: Understand REF: p. 711 TOP: Nursing Process: Planning

12. The client makes an appointment for preconception counseling. The woman has a known heart condition and is unsure if she should become pregnant. Which is the only cardiac condition that would cause concern? a. Marfan syndrome b. Eisenmenger syndrome c. Heart transplant d. Ventricular septal defect (VSD)

: B Pregnancy is contraindicated in clients with Eisenmenger syndrome. Women who have had heart transplants are successfully having babies. However, conception should be postponed for at least 1 year after transplantation. Management of the client with Marfan syndrome during pregnancy includes bed rest, beta-blockers, and surgery before conception. VSD is usually corrected early in life and is therefore not a contraindication to pregnancy. DIF: Cognitive Level: Understand REF: p. 714 TOP: Nursing Process: Assessment

15. Bell palsy is an acute idiopathic facial paralysis, the cause for which remains unknown. Which statement regarding this condition is correct? a. Bell palsy is the sudden development of bilateral facial weakness. b. Women with Bell palsy have an increased risk for hypertension. c. Pregnant women are affected twice as often as nonpregnant women. d. Bell palsy occurs most frequently in the first trimester.

: B The clinical manifestations of Bell palsy include the development of unilateral facial weakness, pain surrounding the ears, difficulty closing the eye, and hyperacusis. The cause is unknown; however, Bell palsy may be related to a viral infection. Pregnant women are affected at a rate of three to five times that of nonpregnant women. The incidence rate peaks during the third trimester and puerperium. Women who develop Bell palsy in pregnancy have an increased risk for hypertension. DIF: Cognitive Level: Understand REF: p. 726 TOP: Nursing Process: Assessment

18. It is extremely rare for a woman to die in childbirth; however, it can happen. In the United States, the annual occurrence of maternal death is 12 per 100,000 cases of live birth. What are the leading causes of maternal death? a. Embolism and preeclampsia b. Trauma and motor vehicle accidents (MVAs) c. Hemorrhage and infection d. Underlying chronic conditions

: B Trauma is the leading cause of obstetric death in women of childbearing age. Most maternal injuries are the result of MVAs and falls. Although preeclampsia and embolism are significant contributors to perinatal morbidity, these are not the leading cause of maternal mortality. Maternal death caused by trauma may occur as the result of hemorrhagic shock or abruptio placentae. In these cases, the hemorrhage is the result of trauma, not childbirth. The wish to become a parent is not eliminated by a chronic health problem, and many women each year risk their lives to have a baby. Because of advanced pediatric care, many women are surviving childhood illnesses and reaching adulthood with chronic health problems such as cystic fibrosis, diabetes, and pulmonary disorders. DIF: Cognitive Level: Understand REF: p. 731 TOP: Nursing Process: Assessment

4. Autoimmune disorders often occur during pregnancy because a large percentage of women with an autoimmune disorder are of childbearing age. Which disorders fall into the category of collagen vascular disease? (Select all that apply.) a. Multiple sclerosis b. SLE c. Antiphospholipid syndrome d. Rheumatoid arthritis e. Myasthenia gravis

: B, C, D, E Multiple sclerosis is not an autoimmune disorder. This patchy demyelination of the spinal cord may be a viral disorder. Autoimmune disorders (collagen vascular disease) make up a large group of conditions that disrupt the function of the immune system of the body. These disorders include those listed, as well as systemic sclerosis. DIF: Cognitive Level: Comprehend REF: p. 726 TOP: Nursing Process: Assessment

6. Which important component of nutritional counseling should the nurse include in health teaching for a pregnant woman who is experiencing cholecystitis? a. Assess the woman's dietary history for adequate calories and proteins. b. Teach the woman that the bulk of calories should come from proteins. c. Instruct the woman to eat a low-fat diet and to avoid fried foods. d. Instruct the woman to eat a low-cholesterol, low-salt diet.

: C Eating a low-fat diet and avoiding fried foods is appropriate nutritional counseling for this client. Caloric and protein intake do not predispose a woman to the development of cholecystitis. The woman should be instructed to limit protein intake and choose foods that are high in carbohydrates. A low-cholesterol diet may be the result of limiting fats. However, a low-salt diet is not indicated. DIF: Cognitive Level: Apply REF: p. 728 TOP: Nursing Process: Implementation

10. A woman at 28 weeks of gestation experiences blunt abdominal trauma as the result of a fall. The nurse must closely observe the client for what? a. Alteration in maternal vital signs, especially blood pressure b. Complaints of abdominal pain c. Placental absorption d. Hemorrhage

: C Electronic fetal monitoring (EFM) tracings can help evaluate maternal status after trauma and can reflect fetal cardiac responses to hypoxia and hypoperfusion. Signs and symptoms of placental absorption include uterine irritability, contractions, vaginal bleeding, and changes in FHR characteristics. Hypoperfusion may be present in the pregnant woman before the onset of clinical signs of shock. EFM tracings show the first signs of maternal compromise, such as when the maternal heart rate, blood pressure, and color appear normal, yet the EFM printout shows signs of fetal hypoxia. Abdominal pain, in and of itself, is not the most important symptom. However, if it is accompanied by contractions, changes in the FHR, rupture of membranes, or vaginal bleeding, then the client should be evaluated for abruptio placentae. Clinical signs of hemorrhage do not appear until after a 30% loss of circulating volume occurs. Careful monitoring of fetal status significantly assists in maternal assessment, because the fetal monitor tracing works as an oximeter of internal well-being. DIF: Cognitive Level: Apply REF: p. 732 TOP: Nursing Process: Assessment

16. A pregnant woman at term is transported to the emergency department (ED) after a severe vehicular accident. The obstetric nurse responds and rushes to the ED with a fetal monitor. Cardiopulmonary arrest occurs as the obstetric nurse arrives. What is the highest priority for the trauma team? a. Obtaining IV access, and starting aggressive fluid resuscitation b. Quickly applying the fetal monitor to determine whether the fetus viability c. Starting cardiopulmonary resuscitation (CPR) d. Transferring the woman to the surgical unit for an emergency cesarean delivery in case the fetus is still alive

: C In a situation of severe maternal trauma, the systematic evaluation begins with a primary survey and the initial ABCs (airway, breathing, and circulation) of resuscitation. CPR is initiated first, followed by intravenous (IV) replacement fluid. After immediate resuscitation and successful stabilization measures, a more detailed secondary survey of the mother and fetus should be accomplished. Attempts at maternal resuscitation are made, followed by a secondary survey of the fetus. In the presence of multisystem trauma, a cesarean delivery may be indicated to increase the chance for maternal survival. DIF: Cognitive Level: Apply REF: p. 734 TOP: Nursing Process: Implementation

5. A woman with asthma is experiencing a postpartum hemorrhage. Which drug should be avoided when treating postpartum bleeding to avoid exacerbating asthma? a. Oxytocin (Pitocin) b. Nonsteroidal antiinflammatory drugs (NSAIDs) c. Hemabate d. Fentanyl

: C Prostaglandin derivatives should not be used to treat women with asthma, because they may exacerbate symptoms. Oxytocin is the drug of choice to treat this woman's bleeding; it will not exacerbate her asthma. NSAIDs are not used to treat bleeding. Fentanyl is used to treat pain, not bleeding. DIF: Cognitive Level: Analyze REF: p. 722 TOP: Nursing Process: Planning

4. A woman has experienced iron deficiency anemia during her pregnancy. She had been taking iron for 3 months before the birth. The client gave birth by cesarean 2 days earlier and has been having problems with constipation. After assisting her back to bed from the bathroom, the nurse notes that the woman's stools are dark (greenish-black). What should the nurse's initial action be? a. Perform a guaiac test, and record the results. b. Recognize the finding as abnormal, and report it to the primary health care provider. c. Recognize the finding as a normal result of iron therapy. d. Check the woman's next stool to validate the observation.

: C The nurse should recognize that dark stools are a common side effect in clients who are taking iron replacement therapy. A guaiac test would be indicated if gastrointestinal (GI) bleeding was suspected. GI irritation, including dark stools, is also a common side effect of iron therapy. Observation of stool formation is a normal nursing activity. DIF: Cognitive Level: Apply REF: p. 716 TOP: Nursing Process: Evaluation

11. Which neurologic condition would require preconception counseling, if at all possible? a. Eclampsia b. Bell palsy c. Epilepsy d. Multiple sclerosis

: C Women with epilepsy should receive preconception counseling, if at all possible. Achieving seizure control before becoming pregnant is a desirable state. Medication should also be carefully reviewed. Eclampsia may sometimes be confused with epilepsy, and Bell palsy is a form of facial paralysis; preconception counseling for either condition is not essential to care. Multiple sclerosis is a patchy demyelination of the spinal cord that does not affect the normal course of pregnancy or birth. DIF: Cognitive Level: Understand REF: p. 725 TOP: Nursing Process: Planning

7. Postoperative care of the pregnant woman who requires abdominal surgery for appendicitis includes which additional assessment? a. Intake and output (I&O) and intravenous (IV) site b. Signs and symptoms of infection c. Vital signs and incision d. Fetal heart rate (FHR) and uterine activity

: D Care of a pregnant woman undergoing surgery for appendicitis differs from that for a nonpregnant woman in one significant aspect: the presence of the fetus. Continuous fetal and uterine monitoring should take place. An assessment of I&O levels, along with an assessment of the IV site, are normal postoperative care procedures. Evaluating the client for signs and symptoms of infection is also part of routine postoperative care. Routine vital signs and evaluation of the incision site are expected components of postoperative care. DIF: Cognitive Level: Apply REF: p. 730 TOP: Nursing Process: Assessment

13. What form of heart disease in women of childbearing years generally has a benign effect on pregnancy? a. Cardiomyopathy b. Rheumatic heart disease c. Congenital heart disease d. Mitral valve prolapse

: D Mitral valve prolapse is a benign condition that is usually asymptomatic. Cardiomyopathy produces congestive heart failure during pregnancy. Rheumatic heart disease can lead to heart failure during pregnancy. Some congenital heart diseases produce pulmonary hypertension or endocarditis during pregnancy. DIF: Cognitive Level: Remember REF: p. 713 TOP: Nursing Process: Assessment

14. A pregnant woman at 33 weeks of gestation is brought to the birthing unit after a minor automobile accident. The client is experiencing no pain and no vaginal bleeding, her vital signs are stable, and the FHR is 132 beats per minute with variability. What is the nurse's highestpriority? a. Monitoring the woman for a ruptured spleen b. Obtaining a physician's order to discharge her home c. Monitoring her for 24 hours d. Using continuous EFM for a minimum of 4 hours

: D Monitoring the external FHR and contractions is recommended after blunt trauma in a viable gestation for a minimum of 4 hours, regardless of injury severity. Fetal monitoring should be initiated as soon as the woman is stable. In this scenario, no clinical findings indicate the possibility of a ruptured spleen. If the maternal and fetal findings are normal, then EFM should continue for a minimum of 4 hours after a minor trauma or a minor automobile accident. Once the monitoring has been completed and the health care provider is reassured of fetal well-being, the client may be discharged home. Monitoring for 24 hours is unnecessary unless the ERM strip is abnormal or nonreassuring. DIF: Cognitive Level: Apply REF: p. 732 TOP: Nursing Process: Planning

17. Another common pregnancy-specific condition is pruritic urticarial papules and plaques of pregnancy (PUPPP). A client asks the nurse why she has developed this condition and what can be done. What is the nurse's bestresponse? a. PUPPP is associated with decreased maternal weight gain. b. The rate of hypertension decreases with PUPPP. c. This common pregnancy-specific condition is associated with a poor fetal outcome. d. The goal of therapy is to relieve discomfort.

: D PUPPP is associated with increased maternal weight gain, increased rate of twin gestation, and hypertension. It is not, however, associated with poor maternal or fetal outcomes. The goal of therapy is simply to relieve discomfort. Antipruritic topical medications, topical steroids, and antihistamines usually provide relief. PUPPP usually resolves before childbirth or shortly thereafter. DIF: Cognitive Level: Apply REF: p. 724 TOP: Nursing Process: Planning

1. When caring for a pregnant woman with cardiac problems, the nurse must be alert for the signs and symptoms of cardiac decompensation. Which critical findings would the nurse find on assessment of the client experiencing this condition? a. Regular heart rate and hypertension b. Increased urinary output, tachycardia, and dry cough c. Shortness of breath, bradycardia, and hypertension d. Dyspnea, crackles, and an irregular, weak pulse

: D Signs of cardiac decompensation include dyspnea; crackles; an irregular, weak, and rapid pulse; rapid respirations; a moist and frequent cough; generalized edema; increasing fatigue; and cyanosis of the lips and nailbeds. A regular heart rate and hypertension are not generally associated with cardiac decompensation. Of the symptoms of increased urinary output, tachycardia, and dry cough, only tachycardia is indicative of cardiac decompensation. Of the symptoms of shortness of breath, bradycardia, and hypertension, only dyspnea is indicative of cardiac decompensation. DIF: Cognitive Level: Understand REF: p. 716 TOP: Nursing Process: Assessment

Which opiate causes euphoria, relaxation, drowsiness, and detachment from reality and has possible effects on the pregnancy, including preeclampsia, intrauterine growth restriction, and premature rupture of membranes? a. Heroin b. Alcohol c. Phencyclidine palmitate (PCP) d. Cocaine

A

15. With one exception, the safest pregnancy is one during which the woman is drug and alcohol free. What is the optimal treatment for women addicted to opioids? a. Methadone maintenance treatment (MMT) b. Detoxification c. Smoking cessation d. 4 Ps Plus

A MMT is currently considered the standard of care for pregnant women who are dependent on heroin or other narcotics. Buprenorphine is another medication approved for the treatment of opioid addiction that is increasingly being used during pregnancy. Opioid replacement therapy has been shown to decrease opioid and other drug use, reduce criminal activity, improve individual functioning, and decrease the rates of infections such as hepatitis B and C, human immunodeficiency virus (HIV), and other STIs. Detoxification is the treatment used for alcohol addiction. Pregnant women requiring withdrawal from alcohol should be admitted for inpatient management. Women are more likely to stop smoking during pregnancy than at any other time in their lives. A smoking cessation program can assist in achieving this goal. The 4 Ps Plus is a screening tool specifically designed to identify pregnant women who need in-depth assessment related to substance abuse.

Screening at 24 weeks of gestation reveals that a pregnant woman has gestational diabetes mellitus (GDM). In planning her care, the nurse and the client mutually agree that an expected outcome is to prevent injury to the fetus as a result of GDM. This fetus is at the greatest risk for which condition? a. Macrosomia b. Congenital anomalies of the central nervous system c. Preterm birth d. Low birth weight

A Poor glycemic control later in pregnancy increases the rate of fetal macrosomia. Poor glycemic control during the preconception time frame and into the early weeks of the pregnancy is associated with congenital anomalies. Preterm labor or birth is more likely to occur with severe diabetes and is the greatest risk in women with pregestational diabetes. Increased weight, or macrosomia, is the greatest risk factor for this fetus.

Which statement concerning the complication of maternal diabetes is the most accurate? a. Diabetic ketoacidosis (DKA) can lead to fetal death at any time during pregnancy. b. Hydramnios occurs approximately twice as often in diabetic pregnancies than in nondiabetic pregnancies. c. Infections occur about as often and are considered about as serious in both diabetic and nondiabetic pregnancies. d. Even mild-to-moderate hypoglycemic episodes can have significant effects on fetal well-being.

A Prompt treatment of DKA is necessary to save the fetus and the mother. Hydramnios occurs 10 times more often in diabetic pregnancies. Infections are more common and more serious in pregnant women with diabetes. Mild-to-moderate hypoglycemic episodes do not appear to have significant effects on fetal well-being.

4. Despite warnings, prenatal exposure to alcohol continues to far exceed exposure to illicit drugs. Which condition is rarely associated with fetal alcohol syndrome (FAS)? a. Respiratory conditions b. Intellectual impairment c. Neural development disorder d. Alcohol-related birth defects (ARBDs)

A Respiratory difficulties are not attributed to exposure to alcohol in utero. Other abnormalities related to FAS include mental retardation, neurodevelopment disorders, and ARBDs.

Which major neonatal complication is carefully monitored after the birth of the infant of a diabetic mother? a. Hypoglycemia b. Hypercalcemia c. Hypobilirubinemia d. Hypoinsulinemia

A The neonate is at highest risk for hypoglycemia because fetal insulin production is accelerated during pregnancy to metabolize excessive glucose from the mother. At birth, the maternal glucose supply stops and the neonatal insulin exceeds the available glucose, thus leading to hypoglycemia. Hypocalcemia is associated with preterm birth, birth trauma, and asphyxia, all common problems of the infant of a diabetic mother. Excess erythrocytes are broken down after birth, and large amounts of bilirubin are released into the neonates circulation, with resulting hyperbilirubinemia. Because fetal insulin production is accelerated during pregnancy, hyperinsulinemia develops in the neonate.

Diabetes refers to a group of metabolic diseases characterized by hyperglycemia resulting from defects in insulin action, insulin secretion, or both. Over time, diabetes causes significant changes in the microvascular and macrovascular circulations. What do these complications include? (Select all that apply.) a. Atherosclerosis b. Retinopathy c. Intrauterine fetal death (IUFD) d. Nephropathy e. Neuropathy f. Autonomic neuropathy

A, B, D, E These structural changes will most likely affect a variety of systems, including the heart, eyes, kidneys, and nerves. IUFD (stillbirth) remains a major complication of diabetes in pregnancy; however, this is a fetal complication.

Possible alternative and complementary therapies for postpartum depression (PPD) for breastfeeding mothers include (Select all that apply): a. Acupressure. b. Aromatherapy. c. St. John's wort. d. Wine consumption. e. Yoga.

A, B, E Possible alternative/complementary therapies for postpartum depression include acupuncture, acupressure, aromatherapy, therapeutic touch, massage, relaxation techniques, reflexology, and yoga. St. John's wort has not been proven to be safe for women who are breastfeeding. Women who are breastfeeding and/or have a history of PPD should not consume alcohol.

A serious but uncommon complication of undiagnosed or partially treated hyperthyroidism is a thyroid storm, which may occur in response to stress such as infection, birth, or surgery. What are the signs and symptoms of this emergency disorder? (Select all that apply.) a. Fever b. Hypothermia c. Restlessness d. Bradycardia e. Hypertension

A, C Fever, restlessness, tachycardia, vomiting, hypotension, and stupor are symptoms of a thyroid storm. Fever, not hypothermia; tachycardia, not bradycardia; and hypotension, not hypertension, are symptoms of thyroid storm.

23. A client at 39 weeks of gestation has been admitted for an external version. Which intervention would the nurse anticipate the provider to order? a. Tocolytic drug b. Contraction stress test (CST) c. Local anesthetic d. Foley catheter

ANS: A A tocolytic drug will relax the uterus before and during the version, thus making manipulation easier. CST is used to determine the fetal response to stress. A local anesthetic is not used with external version. Although the bladder should be emptied, catheterization is not necessary. DIF: Cognitive Level: Apply REF: p. 779 TOP: Nursing Process: Planning MSC: Client Needs: Physiologic Integrity

4. A woman in preterm labor at 30 weeks of gestation receives two 12-mg intramuscular (IM) doses of betamethasone. What is the purpose of this pharmacologic intervention? a. To stimulate fetal surfactant production b. To reduce maternal and fetal tachycardia associated with ritodrine administration c. To suppress uterine contractions d. To maintain adequate maternal respiratory effort and ventilation during magnesium sulfate therapy

ANS: A Antenatal glucocorticoids administered as IM injections to the mother accelerate fetal lung maturity. Propranolol (Inderal) is given to reduce the effects of ritodrine administration. Betamethasone has no effect on uterine contractions. Calcium gluconate is given to reverse the respiratory depressive effects of magnesium sulfate therapy. DIF: Cognitive Level: Understand REF: p. 769 TOP: Nursing Process: Planning MSC: Client Needs: Physiologic Integrity

21. What is the highest priority nursing intervention when admitting a pregnant woman who has experienced a bleeding episode in late pregnancy? a. Assessing FHR and maternal vital signs b. Performing a venipuncture for hemoglobin and hematocrit levels c. Placing clean disposable pads to collect any drainage d. Monitoring uterine contractions

ANS: A Assessment of the FHR and maternal vital signs will assist the nurse in determining the degree of the blood loss and its effect on the mother and fetus. The most important assessment is to check the well-being of both the mother and the fetus. The blood levels can be obtained later. Assessing future bleeding is important; however, the top priority remains mother/fetal well-being. Monitoring uterine contractions is important but not a top priority. DIF: Cognitive Level: Apply REF: p. 681 TOP: Nursing Process: Implementation MSC: Client Needs: Health Promotion and Maintenance

15. Which description most accurately describes the augmentation of labor? a. Is part of the active management of labor that is instituted when the labor process is unsatisfactory b. Relies on more invasive methods when oxytocin and amniotomy have failed c. Is a modern management term to cover up the negative connotations of forceps-assisted birth d. Uses vacuum cups

ANS: A Augmentation is part of the active management of labor that stimulates uterine contractions after labor has started but is not progressing satisfactorily. Augmentation uses amniotomy and oxytocin infusion, as well as some more gentle, noninvasive methods. Forceps-assisted births are less common than in the past and not considered a method of augmentation. A vacuum-assisted delivery occurs during childbirth if the mother is too exhausted to push. Vacuum extraction is not considered an augmentation methodology. DIF: Cognitive Level: Understand REF: pp. 785-786 TOP: Nursing Process: Planning MSC: Client Needs: Health Promotion and Maintenance

22. Which order should the nurse expect for a client admitted with a threatened abortion? a. Bed rest b. Administration of ritodrine IV c. Nothing by mouth (nil per os [NPO]) d. Narcotic analgesia every 3 hours, as needed

ANS: A Decreasing the woman's activity level may alleviate the bleeding and allow the pregnancy to continue. Ritodrine is not the first drug of choice for tocolytic medications. Having the woman placed on NPO is unnecessary. At times, dehydration may produce contractions; therefore, hydration is important. Narcotic analgesia will not decrease the contractions and may mask the severity of the contractions. DIF: Cognitive Level: Understand REF: pp. 671-672 TOP: Nursing Process: Planning MSC: Client Needs: Health Promotion and Maintenance

16. The exact cause of preterm labor is unknown but believed to be multifactorial. Infection is thought to be a major factor in many preterm labors. Which type of infection has not been linked to preterm birth? a. Viral b. Periodontal c. Cervical d. Urinary tract

ANS: A Infections that increase the risk of preterm labor and birth are bacterial and include cervical, urinary tract, periodontal, and other bacterial infections. Therefore, early, continual, and comprehensive participation by the client in her prenatal care is important. Recent evidence has shown a link between periodontal infections and preterm labor. Researchers recommend regular dental care before and during pregnancy, oral assessment as a routine part of prenatal care, and scrupulous oral hygiene to prevent periodontal infections. DIF: Cognitive Level: Remember REF: p. 760 TOP: Nursing Process: Assessment MSC: Client Needs: Physiologic Integrity

25. Which nursing intervention should be immediately performed after the forceps-assisted birth of an infant? a. Assessing the infant for signs of trauma b. Administering prophylactic antibiotic agents to the infant c. Applying a cold pack to the infant's scalp d. Measuring the circumference of the infant's head

ANS: A The infant should be assessed for bruising or abrasions at the site of application, facial palsy, and subdural hematoma. Prophylactic antibiotics are not necessary with a forceps delivery. A cold pack would place the infant at risk for cold stress and is contraindicated. Measuring the circumference of the head is part of the initial nursing assessment. DIF: Cognitive Level: Apply REF: p. 788 TOP: Nursing Process: Implementation MSC: Client Needs: Physiologic Integrity

20. A pregnant woman's amniotic membranes have ruptured. A prolapsed umbilical cord is suspected. What intervention would be the nurse's highest priority? a. Placing the woman in the knee-chest position b. Covering the cord in sterile gauze soaked in saline c. Preparing the woman for a cesarean birth d. Starting oxygen by face mask

ANS: A The woman is assisted into a modified Sims position, Trendelenburg position, or the knee-chest position in which gravity keeps the pressure of the presenting part off the cord. Although covering the cord in sterile gauze soaked saline, preparing the woman for a cesarean, and starting oxygen by face mark are appropriate nursing interventions in the event of a prolapsed cord, the intervention of top priority would be positioning the mother to relieve cord compression. DIF: Cognitive Level: Apply REF: p. 797 TOP: Nursing Process: Implementation MSC: Client Needs: Physiologic Integrity

6. According to research, which risk factor for PPD is likely to have the greatest effect on the client postpartum? a. Prenatal depression b. Single-mother status c. Low socioeconomic status d. Unplanned or unwanted pregnancy

ANS: A Prenatal depression has been found to be a major risk factor for PPD. Single-mother status and low socioeconomic status are both small-relationship predictors for PPD. Although an unwanted pregnancy may contribute to the risk for PPD, it does not pose as great an effect as prenatal depression.

2. What are the complications and risks associated with cesarean births? (Select all that apply.) a. Pulmonary edema b. Wound dehiscence c. Hemorrhage d. Urinary tract infections e. Fetal injuries

ANS: A, B, C, D, E Pulmonary edema, wound dehiscence, hemorrhage, urinary tract infections, and fetal injuries are possible complications and risks associated with cesarean births. DIF: Cognitive Level: Understand REF: p. 790 TOP: Nursing Process: Evaluation MSC: Client Needs: Physiologic Integrity

3. Women who are obese are at risk for several complications during pregnancy and birth. Which of these would the nurse anticipate with an obese client? (Select all that apply.) a. Thromboembolism b. Cesarean birth c. Wound infection d. Breech presentation e. Hypertension

ANS: A, B, C, E A breech presentation is not a complication of pregnancy or birth for the client who is obese. Venous thromboembolism is a known risk for obese women. Therefore, the use of thromboembolism-deterrent (TED) hose and sequential compression devices may help decrease the chance for clot formation. Women should also be encouraged to ambulate as soon as possible. In addition to having an increased risk for complications with a cesarean birth, in general, obese women are also more likely to require an emergency cesarean birth. Many obese women have a pannus (i.e., large roll of abdominal fat) that overlies a lower transverse incision made just above the pubic area. The pannus causes the area to remain moist, which encourages the development of infection. Obese women are more likely to begin pregnancy with comorbidities such as hypertension and type 2 diabetes. DIF: Cognitive Level: Analyze REF: p. 778 TOP: Nursing Process: Assessment MSC: Client Needs: Health Promotion and Maintenance

3. The reported incidence of ectopic pregnancy has steadily risen over the past 2 decades. Causes include the increase in sexually transmitted infections (STIs) accompanied by tubal infection and damage. The popularity of contraceptive devices such as the IUD has also increased the risk for ectopic pregnancy. The nurse suspects that a client has early signs of ectopic pregnancy. The nurse should be observing the client for which signs or symptoms? (Select all that apply.) a. Pelvic pain b. Abdominal pain c. Unanticipated heavy bleeding d. Vaginal spotting or light bleeding e. Missed period

ANS: A, B, D, E A missed period or spotting can be easily mistaken by the client as an early sign of pregnancy. More subtle signs depend on exactly where the implantation occurs. The nurse must be thorough in her assessment because pain is not a normal symptom of early pregnancy. As the fallopian tube tears open and the embryo is expelled, the client often exhibits severe pain accompanied by intraabdominal hemorrhage, which may progress to hypovolemic shock with minimal or even no external bleeding. In approximately one half of women, shoulder and neck pain results from irritation of the diaphragm from the hemorrhage. DIF: Cognitive Level: Apply REF: p. 676 TOP: Nursing Process: Assessment MSC: Client Needs: Physiologic Integrity

2. Approximately 10% to 15% of all clinically recognized pregnancies end in miscarriage. What are possible causes of early miscarriage? (Select all that apply.) a. Chromosomal abnormalities b. Infections c. Endocrine imbalance d. Systemic disorders e. Varicella

ANS: A, C, D, E Infections are not a common cause of early miscarriage. At least 50% of pregnancy losses result from chromosomal abnormalities. Endocrine imbalances such as hypothyroidism or diabetes are also possible causes for early pregnancy loss. Other systemic disorders that may contribute to pregnancy loss include lupus and genetic conditions. Although infections are not a common cause of early miscarriage, varicella infection in the first trimester has been associated with pregnancy loss. DIF: Cognitive Level: Remember REF: p. 669 TOP: Nursing Process: Assessment MSC: Client Needs: Health Promotion and Maintenance

4. The induction of labor is considered an acceptable obstetric procedure if it is in the best interest to deliver the fetus. The charge nurse on the labor and delivery unit is often asked to schedule clients for this procedure and therefore must be cognizant of the specific conditions appropriate for labor induction. What are appropriate indications for induction? (Select all that apply?) a. Rupture of membranes at or near term b. Convenience of the woman or her physician c. Chorioamnionitis (inflammation of the amniotic sac) d. Postterm pregnancy e. Fetal death

ANS: A, C, D, E The conditions listed are all acceptable indications for induction. Other conditions include intrauterine growth restriction (IUGR), maternal-fetal blood incompatibility, hypertension, and placental abruption. Elective inductions for the convenience of the woman or her provider are not recommended; however, they have become commonplace. Factors such as rapid labors and living a long distance from a health care facility may be valid reasons in such a circumstance. Elective delivery should not occur before 39 weeks of completed gestation. DIF: Cognitive Level: Apply REF: p. 790 TOP: Nursing Process: Planning MSC: Client Needs: Physiologic Integrity

14. A number of methods can be used for inducing labor. Which cervical ripening method falls under the category of mechanical or physical? a. Prostaglandins are used to soften and thin the cervix. b. Labor can sometimes be induced with balloon catheters or laminaria tents. c. Oxytocin is less expensive and more effective than prostaglandins but creates greater health risks. d. Amniotomy can be used to make the cervix more favorable for labor.

ANS: B Balloon catheters or laminaria tents are mechanical means of ripening the cervix. Ripening the cervix, making it softer and thinner, increases the success rate of induced labor. Prostaglandin E1 is less expensive and more effective than oxytocin but carries a greater risk. Amniotomy is the artificial rupture of membranes, which is used to induce labor only when the cervix is already ripe. DIF: Cognitive Level: Apply REF: p. 781 TOP: Nursing Process: Planning MSC: Client Needs: Health Promotion and Maintenance

10. The nurse who elects to work in the specialty of obstetric care must have the ability to distinguish between preterm birth, preterm labor, and low birth weight. Which statement regarding this terminology is correct? a. Terms preterm birth and low birth weight can be used interchangeably. b. Preterm labor is defined as cervical changes and uterine contractions occurring between 20 and 37 weeks of gestation. c. Low birth weight is a newborn who weighs below 3.7 pounds. d. Preterm birth rate in the United States continues to increase.

ANS: B Before 20 weeks of gestation, the fetus is not viable (miscarriage); after 37 weeks, the fetus can be considered term. Although these terms are used interchangeably, they have different meanings: preterm birth describes the length of gestation (before 37 weeks), regardless of the newborn's weight; low birth weight describes only the infant's weight at the time of birth (2500 g or less), whenever it occurs. Low birth weight is anything below 2500 g or approximately pounds. In 2011, the preterm birth rate in the United States was 11.7 %; it has dropped every year since 2008. DIF: Cognitive Level: Understand REF: p. 759 TOP: Nursing Process: Assessment MSC: Client Needs: Health Promotion and Maintenance

19. What is the correct terminology for an abortion in which the fetus dies but is retained within the uterus? a. Inevitable abortion b. Missed abortion c. Incomplete abortion d. Threatened abortion

ANS: B Missed abortion refers to the retention of a dead fetus in the uterus. An inevitable abortion means that the cervix is dilating with the contractions. An incomplete abortion means that not all of the products of conception were expelled. With a threatened abortion, the woman has cramping and bleeding but no cervical dilation. DIF: Cognitive Level: Remember REF: p. 670 TOP: Nursing Process: Assessment MSC: Client Needs: Physiologic Integrity

2. Which nursing intervention is paramount when providing care to a client with preterm labor who has received terbutaline? a. Assess deep tendon reflexes (DTRs). b. Assess for dyspnea and crackles. c. Assess for bradycardia. d. Assess for hypoglycemia.

ANS: B Terbutaline is a beta2-adrenergic agonist that affects the mother's cardiopulmonary and metabolic systems. Signs of cardiopulmonary decompensation include adventitious breath sounds and dyspnea. An assessment for dyspnea and crackles is important for the nurse to perform if the woman is taking magnesium sulfate. Assessing DTRs does not address the possible respiratory side effects of using terbutaline. Since terbutaline is a beta2-adrenergic agonist, it can lead to hyperglycemia, not hypoglycemia. Beta2-adrenergic agonist drugs cause tachycardia, not bradycardia. DIF: Cognitive Level: Analyze REF: pp. 767-768 TOP: Nursing Process: Assessment MSC: Client Needs: Physiologic Integrity

8. A pregnant woman at 29 weeks of gestation has been diagnosed with preterm labor. Her labor is being controlled with tocolytic medications. She asks when she might be able to go home. Which response by the nurse is mostaccurate? a. "After the baby is born." b. "When we can stabilize your preterm labor and arrange home health visits." c. "Whenever your physician says that it is okay." d. "It depends on what kind of insurance coverage you have."

ANS: B This client's preterm labor is being controlled with tocolytics. Once she is stable, home care may be a viable option for this type of client. Care of a client with preterm labor is multidisciplinary and multifactorial; the goal is to prevent delivery. In many cases, this goal may be achieved at home. Managed care may dictate an earlier hospital discharge or a shift from hospital to home care. Insurance coverage may be one factor in client care, but ultimately, client safety remains the most important factor. DIF: Cognitive Level: Apply REF: p. 765 TOP: Nursing Process: Planning MSC: Client Needs: Health Promotion and Maintenance

5. Indications for a primary cesarean birth are often nonrecurring. Therefore, a woman who has had a cesarean birth with a low transverse scar may be a candidate for vaginal birth after cesarean (VBAC). Which clients would beless likely to have a successful VBAC? (Select all that apply.) a. Lengthy interpregnancy interval b. African-American race c. Delivery at a rural hospital d. Estimated fetal weight <4000 g e. Maternal obesity (BMI >30)

ANS: B, C, E Indications for a low success rate for a VBAC delivery include a short interpregnancy interval, non-Caucasian race, gestational age longer than 40 weeks, maternal obesity, preeclampsia, fetal weight greater than 4000 g, and delivery at a rural or private hospital. DIF: Cognitive Level: Understand REF: p. 794 TOP: Nursing Process: Assessment MSC: Client Needs: Physiologic Integrity

MULTIPLE RESPONSE 1. The nurse recognizes that uterine hyperstimulation with oxytocin requires emergency interventions. What clinical cues alert the nurse that the woman is experiencing uterine hyperstimulation? (Select all that apply.) a. Uterine contractions lasting <90 seconds and occurring >2 minutes in frequency b. Uterine contractions lasting >90 seconds and occurring <2 minutes in frequency c. Uterine tone <20 mm Hg d. Uterine tone >20 mm Hg e. Increased uterine activity accompanied by a nonreassuring FHR and pattern

ANS: B, D, E Uterine contractions that occur less frequently than 2 minutes apart and last longer than 90 seconds, a uterine tone over 20 mm Hg, and a nonreassuring FHR and pattern are indications of uterine hyperstimulation with oxytocin administration. Uterine contractions that occur more frequently than 2 minutes apart and last less than 90 seconds are the expected goal of oxytocin induction. A uterine tone less than 20 mm Hg is normal. DIF: Cognitive Level: Analyze REF: p. 785 TOP: Nursing Process: Implementation MSC: Client Needs: Physiologic Integrity

21. What is the primary purpose for the use of tocolytic therapy to suppress uterine activity? a. Drugs can be efficaciously administered up to the designated beginning of term at 37 weeks gestation. b. Tocolytic therapy has no important maternal (as opposed to fetal) contraindications. c. The most important function of tocolytic therapy is to provide the opportunity to administer antenatal glucocorticoids. d. If the client develops pulmonary edema while receiving tocolytic therapy, then intravenous (IV) fluids should be given.

ANS: C Buying time for antenatal glucocorticoids to accelerate fetal lung development may be the best reason to use tocolytic therapy. Once the pregnancy has reached 34 weeks, however, the risks of tocolytic therapy outweigh the benefits. Important maternal contraindications to tocolytic therapy exist. Tocolytic-induced edema can be caused by IV fluids. DIF: Cognitive Level: Comprehend REF: p. 766 TOP: Nursing Process: Planning MSC: Client Needs: Physiologic Integrity

20. What condition indicates concealed hemorrhage when the client experiences abruptio placentae? a. Decrease in abdominal pain b. Bradycardia c. Hard, boardlike abdomen d. Decrease in fundal height

ANS: C Concealed hemorrhage occurs when the edges of the placenta do not separate. The formation of a hematoma behind the placenta and subsequent infiltration of the blood into the uterine muscle results in a very firm, boardlike abdomen. Abdominal pain may increase. The client will have shock symptoms that include tachycardia. As bleeding occurs, the fundal height increases. DIF: Cognitive Level: Analyze REF: p. 683 TOP: Nursing Process: Assessment MSC: Client Needs: Physiologic Integrity

7. Prostaglandin gel has been ordered for a pregnant woman at 43 weeks of gestation. What is the primary purpose of prostaglandin administration? a. To enhance uteroplacental perfusion in an aging placenta b. To increase amniotic fluid volume c. To ripen the cervix in preparation for labor induction d. To stimulate the amniotic membranes to rupture

ANS: C Preparations of prostaglandin E1 and E2 are effective when used before labor induction to ripen (i.e., soften and thin) the cervix. Uteroplacental perfusion is not altered by the use of prostaglandins. The insertion of prostaglandin gel has no effect on the level of amniotic fluid. In some cases, women will spontaneously begin laboring after the administration of prostaglandins, thereby eliminating the need for oxytocin. It is not common for a woman's membranes to rupture as a result of prostaglandin use. DIF: Cognitive Level: Apply REF: p. 779 TOP: Nursing Process: Planning MSC: Client Needs: Physiologic Integrity

1. In planning for home care of a woman with preterm labor, which concern should the nurse need to address? a. Nursing assessments are different from those performed in the hospital setting. b. Restricted activity and medications are necessary to prevent a recurrence of preterm labor. c. Prolonged bed rest may cause negative physiologic effects. d. Home health care providers are necessary.

ANS: C Prolonged bed rest may cause adverse effects such as weight loss, loss of appetite, muscle wasting, weakness, bone demineralization, decreased cardiac output, risk for thrombophlebitis, alteration in bowel functions, sleep disturbance, and prolonged postpartum recovery. Nursing assessments differ somewhat from those performed in the acute care setting, but this concern does not need to be addressed. Restricted activity and medications may prevent preterm labor but not in all women. In addition, the plan of care is individualized to meet the needs of each client. Many women receive home health nurse visits, but care is individualized for each woman. DIF: Cognitive Level: Analyze REF: p. 777 TOP: Nursing Process: Planning MSC: Client Needs: Health Promotion and Maintenance

17. The nurse is teaching a client with preterm premature rupture of membranes (PPROM) regarding self-care activities. Which activities should the nurse include in her teaching? a. Report a temperature higher than 40° C. b. Tampons are safe to use to absorb the leaking amniotic fluid. c. Do not engage in sexual activity. d. Taking frequent tub baths is safe.

ANS: C Sexual activity should be avoided because it may induce preterm labor. A temperature higher than 38° C should be reported. To prevent the risk of infection, tub baths should be avoided and nothing should be inserted into the vagina. Further, foul-smelling vaginal fluid, which may be a sign of infection, should be reported. DIF: Cognitive Level: Apply REF: p. 762 TOP: Nursing Process: Implementation MSC: Client Needs: Health Promotion and Maintenance

5. A primigravida at 40 weeks of gestation is having uterine contractions every to 2 minutes and states that they are very painful. Her cervix is dilated 2 cm and has not changed in 3 hours. The woman is crying and wants an epidural. What is the likely status of this woman's labor? a. She is exhibiting hypotonic uterine dysfunction. b. She is experiencing a normal latent stage. c. She is exhibiting hypertonic uterine dysfunction. d. She is experiencing precipitous labor.

ANS: C The contraction pattern observed in this woman signifies hypertonic uterine activity. Typically, uterine activity in this phase occurs at 4- to 5-minute intervals lasting 30 to 45 seconds. Women who experience hypertonic uterine dysfunction, or primary dysfunctional labor, are often anxious first-time mothers who are having painful and frequent contractions that are ineffective at causing cervical dilation or effacement to progress. With hypotonic uterine dysfunction, the woman initially makes normal progress into the active stage of labor; then the contractions become weak and inefficient or stop altogether. Precipitous labor is one that lasts less than 3 hours from the onset of contractions until time of birth. DIF: Cognitive Level: Apply REF: p. 773 TOP: Nursing Process: Diagnosis MSC: Client Needs: Health Promotion and Maintenance

23. Which finding on a prenatal visit at 10 weeks of gestation might suggest a hydatidiform mole? a. Complaint of frequent mild nausea b. Blood pressure of 120/80 mm Hg c. Fundal height measurement of 18 cm d. History of bright red spotting for 1 day, weeks ago

ANS: C The uterus in a hydatidiform molar pregnancy is often larger than would be expected on the basis of the duration of the pregnancy. Nausea increases in a molar pregnancy because of the increased production of hCG. A woman with a molar pregnancy may have early-onset pregnancy-induced hypertension. In the client's history, bleeding is normally described as brownish. DIF: Cognitive Level: Analyze REF: p. 678 TOP: Nursing Process: Assessment MSC: Client Needs: Health Promotion and Maintenance

12. Which statement related to cephalopelvic disproportion (CPD) is the least accurate? a. CPD can be related to either fetal size or fetal position. b. The fetus cannot be born vaginally. c. CPD can be accurately predicted. d. Causes of CPD may have maternal or fetal origins.

ANS: C Unfortunately, accurately predicting CPD is not possible. Although CPD is often related to excessive fetal size (macrosomia), malposition of the fetal presenting part is the problem in many cases, not true CPD. When CPD is present, the fetus cannot fit through the maternal pelvis to be born vaginally. CPD may be related to either fetal origins such as macrosomia or malposition or maternal origins such as a too small or malformed pelvis. DIF: Cognitive Level: Understand REF: p. 775 TOP: Nursing Process: Planning MSC: Client Needs: Health Promotion and Maintenance

6. A woman is having her first child. She has been in labor for 15 hours. A vaginal examination performed 2 hours earlier revealed the cervix to be dilated to 5 cm and 100% effaced, and the presenting part of the fetus was at station 0; however, another vaginal examination performed 5 minutes ago indicated no changes. What abnormal labor pattern is associated with this description? a. Prolonged latent phase b. Protracted active phase c. Secondary arrest d. Protracted descent

ANS: C With a secondary arrest of the active phase, the progress of labor has stopped. This client has not had any anticipated cervical change, indicating an arrest of labor. In the nulliparous woman, a prolonged latent phase typically lasts longer than 20 hours. A protracted active phase, the first or second stage of labor, is prolonged (slow dilation). With a protracted descent, the fetus fails to descend at an anticipated rate during the deceleration phase and second stage of labor. DIF: Cognitive Level: Analyze REF: p. 774 TOP: Nursing Process: Assessment MSC: Client Needs: Health Promotion and Maintenance

18. A woman at 26 weeks of gestation is being assessed to determine whether she is experiencing preterm labor. Which finding indicates that preterm labor is occurring? a. Estriol is not found in maternal saliva. b. Irregular, mild uterine contractions are occurring every 12 to 15 minutes. c. Fetal fibronectin is present in vaginal secretions. d. The cervix is effacing and dilated to 2 cm.

ANS: D Cervical changes such as shortened endocervical length, effacement, and dilation are predictors of imminent preterm labor. Changes in the cervix accompanied by regular contractions indicate labor at any gestation. Estriol is a form of estrogen produced by the fetus that is present in plasma at 9 weeks of gestation. Levels of salivary estriol have been shown to increase before preterm birth. Irregular, mild contractions that do not cause cervical change are not considered a threat. The presence of fetal fibronectin in vaginal secretions between 24 and 36 weeks of gestation could predict preterm labor, but it has only a 20% to 40% positive predictive value. Of more importance are other physiologic clues of preterm labor such as cervical changes. DIF: Cognitive Level: Apply REF: p. 759 TOP: Nursing Process: Assessment | Nursing Process: Planning MSC: Client Needs: Health Promotion and Maintenance

11. The nurse is performing an assessment on a client who thinks she may be experiencing preterm labor. Which information is the most important for the nurse to understand and share with the client? a. Because all women must be considered at risk for preterm labor and prediction is so variable, teaching pregnant women the symptoms of preterm labor probably causes more harm through false alarms. b. Braxton Hicks contractions often signal the onset of preterm labor. c. Because preterm labor is likely to be the start of an extended labor, a woman with symptoms can wait several hours before contacting the primary caregiver. d. Diagnosis of preterm labor is based on gestational age, uterine activity, and progressive cervical change.

ANS: D Gestational age of 20 to 37 weeks, uterine contractions, and a cervix that is 80% effaced or dilated 2 cm indicates preterm labor. It is essential that nurses teach women how to detect the early symptoms of preterm labor. Braxton Hicks contractions resemble preterm labor contractions, but they are not true labor. Waiting too long to see a health care provider could result in essential medications failing to be administered. Preterm labor is not necessarily long-term labor. DIF: Cognitive Level: Understand REF: p. 759 TOP: Nursing Process: Planning MSC: Client Needs: Safe and Effective Care Environment

13. Which statement related to the induction of labor is most accurate? a. Can be achieved by external and internal version techniques b. Is also known as a trial of labor (TOL) c. Is almost always performed for medical reasons d. Is rated for viability by a Bishop score

ANS: D Induction of labor is likely to be more successful with a Bishop score of 9 or higher for first-time mothers or 5 or higher for veterans. Version is the turning of the fetus to a better position by a physician for an easier or safer birth. A TOL is the observance of a woman and her fetus for several hours of active labor to assess the safety of vaginal birth. Two thirds of cases of induced labor are elective and not done for medical reasons. DIF: Cognitive Level: Understand REF: p. 780 TOP: Nursing Process: Diagnosis MSC: Client Needs: Safe and Effective Care Environment

22. When would an internal version be indicated to manipulate the fetus into a vertex position? a. Fetus from a breech to a cephalic presentation before labor begins b. Fetus from a transverse lie to a longitudinal lie before a cesarean birth c. Second twin from an oblique lie to a transverse lie before labor begins d. Second twin from a transverse lie to a breech presentation during a vaginal birth

ANS: D Internal version is used only during a vaginal birth to manipulate the second twin into a presentation that allows it to be vaginally born. For internal version to occur, the cervix needs to be completely dilated. DIF: Cognitive Level: Remember REF: p. 779 TOP: Nursing Process: Assessment MSC: Client Needs: Physiologic Integrity

19. Which assessment is least likely to be associated with a breech presentation? a. Meconium-stained amniotic fluid b. Fetal heart tones heard at or above the maternal umbilicus c. Preterm labor and birth d. Postterm gestation

ANS: D Postterm gestation is not likely to occur with a breech presentation. The presence of meconium in a breech presentation may be a result of pressure on the fetal wall as it traverses the birth canal. Fetal heart tones heard at the level of the umbilical level of the mother are a typical finding in a breech presentation because the fetal back would be located in the upper abdominal area. Breech presentations often occur in preterm births. DIF: Cognitive Level: Analyze REF: pp. 775-776 TOP: Nursing Process: Assessment MSC: Client Needs: Health Promotion and Maintenance

24. A 32-year-old primigravida is admitted with a diagnosis of ectopic pregnancy. Which information assists the nurse in developing the plan of care? a. Bed rest and analgesics are the recommended treatment. b. She will be unable to conceive in the future. c. A D&C will be performed to remove the products of conception. d. Hemorrhage is the primary concern.

ANS: D Severe bleeding occurs if the fallopian tube ruptures. The recommended treatment is to remove the pregnancy before rupture to prevent hemorrhaging. If the tube must be removed, then the woman's fertility will decrease; however, she will not be infertile. A D&C is performed on the inside of the uterine cavity. The ectopic pregnancy is located within the tubes. DIF: Cognitive Level: Apply REF: p. 676 TOP: Nursing Process: Planning MSC: Client Needs: Physiologic Integrity

9. The obstetric provider has informed the nurse that she will be performing an amniotomy on the client to induce labor. What is the nurse's highest priority intervention after the amniotomy is performed? a. Applying clean linens under the woman b. Taking the client's vital signs c. Performing a vaginal examination d. Assessing the fetal heart rate (FHR)

ANS: D The FHR is assessed before and immediately after the amniotomy to detect any changes that might indicate cord compression or prolapse. Providing comfort measures, such as clean linens, for the client is important but not the priority immediately after an amniotomy. The woman's temperature should be checked every 2 hours after the rupture of membranes but not the priority immediately after an amniotomy. The woman would have had a vaginal examination during the procedure. Unless cord prolapse is suspected, another vaginal examination is not warranted. Additionally, FHR assessment provides clinical cues to a prolapsed cord. DIF: Cognitive Level: Analyze REF: p. 783 TOP: Nursing Process: Implementation MSC: Client Needs: Physiologic Integrity

3. In evaluating the effectiveness of magnesium sulfate for the treatment of preterm labor, which finding alerts the nurse to possible side effects? a. Urine output of 160 ml in 4 hours b. DTRs 2+ and no clonus c. Respiratory rate (RR) of 16 breaths per minute d. Serum magnesium level of 10 mg/dl

ANS: D The therapeutic range for magnesium sulfate management is 4 to 7.5 mg/dl. A serum magnesium level of 10 mg/dl could lead to signs and symptoms of magnesium toxicity, including oliguria and respiratory distress. Urine output of 160 ml in 4 hours, DTRs of 2+, and a RR of 16 breaths per minute are all normal findings. DIF: Cognitive Level: Apply REF: p. 767 TOP: Nursing Process: Evaluation MSC: Client Needs: Physiologic Integrity

5. As a powerful central nervous system (CNS) stimulant, which of these substances can lead to miscarriage, preterm labor, placental separation (abruption), and stillbirth? a. Heroin b. Alcohol c. Phencyclidine (1-phenylcyclohexylpiperidine; PCP) d. Cocaine

ANS: D Cocaine is a powerful CNS stimulant. Effects on pregnancy associated with cocaine use include abruptio placentae, preterm labor, precipitous birth, and stillbirth. Heroin is an opiate; its use in pregnancy is associated with preeclampsia, intrauterine growth restriction, miscarriage, premature rupture of membranes, infections, breech presentation, and preterm labor. The most serious effect of alcohol use in pregnancy is FAS. The major concern regarding PCP use in pregnant women is its association with polydrug abuse and its neurobehavioral effects on the neonate.

The 5 As screening intervention tool is used to implement smoking cessation strategies on the basis of patient response. What do the 5 As stand for? a. Ask, advise, administer, approve, and admit b. Ask, assess, advise, assist, and arrange follow-up c. Assess, assist, advise, apply, and arrange d. Assess, apply, advise, ask, and arrange follow-up

B

1. When caring for a pregnant woman with cardiac problems, the nurse must be alert for the signs and symptoms of cardiac decompensation. Which critical findings would the nurse find on assessment of the client experiencing this condition? a. Regular heart rate and hypertension b. Increased urinary output, tachycardia, and dry cough c. Shortness of breath, bradycardia, and hypertension d. Dyspnea, crackles, and an irregular, weak pulse

B An increased appetite and a lack of interest would reassure the nurse that the client is not experiencing an episode of mania. Clinical manifestations of a manic episode include at least three of the following: grandiosity, decreased need for sleep, pressured speech, flight of ideas, distractibility, psychomotor agitation, and excessive involvement in pleasurable activities. The pregnant woman exhibiting symptoms of a manic episode will likely have a decreased interest in eating and an increased level of interest in pleasurable activities without regard for negative consequences. Psychomotor agitation and a lack of sleep, hyperactivity and distractibility, and pressured speech and grandiosity are all clinical manifestations of a manic episode.

An 18-year-old client who has reached 16 weeks of gestation was recently diagnosed with pregestational diabetes. She attends her centering appointment accompanied by one of her girlfriends. This young woman appears more concerned about how her pregnancy will affect her social life than her recent diagnosis of diabetes. A number of nursing diagnoses are applicable to assist in planning adequate care. What is the most appropriate diagnosis at this time? a. Risk for injury, to the fetus related to birth trauma b. Deficient knowledge, related to diabetic pregnancy management c. Deficient knowledge, related to insulin administration d. Risk for injury, to the mother related to hypoglycemia or hyperglycemia

B Before a treatment plan is developed or goals for the outcome of care are outlined, this client must come to an understanding of diabetes and the potential effects on her pregnancy. She appears more concerned about changes to her social life than adopting a new self-care regimen. Risk for injury to the fetus related to either placental insufficiency or birth trauma may come later in the pregnancy. At this time, the client is having difficulty acknowledging the adjustments that she needs to make to her lifestyle to care for herself during pregnancy. The client may not yet be on insulin. Insulin requirements increase with gestation. The importance of glycemic control must be part of health teaching for this client. However, she has not yet acknowledged that changes to her lifestyle need to be made and may not participate in the plan of care until understanding takes place.

A client with maternal phenylketonuria (PKU) has come to the obstetrical clinic to begin prenatal care. Why would this preexisting condition result in the need for closer monitoring during pregnancy? a. PKU is a recognized cause of preterm labor. b. The fetus may develop neurologic problems. c. A pregnant woman is more likely to die without strict dietary control. d. Women with PKU are usually mentally handicapped and should not reproduce.

B Children born to women with untreated PKU are more likely to be born with mental retardation, microcephaly, congenital heart disease, and low birth weight. Maternal PKU has no effect on labor. Women without dietary control of PKU are more likely to miscarry or bear a child with congenital anomalies. Screening for undiagnosed maternal PKU at the first prenatal visit may be warranted, especially in individuals with a family history of the disorder, with low intelligence of an uncertain cause, or who have given birth to microcephalic infants.

The nurse who is caring for a woman hospitalized for hyperemesis gravidarum would expect the initial treatment to involve what? a. Corticosteroids to reduce inflammation b. Intravenous (IV) therapy to correct fluid and electrolyte imbalances c. Antiemetic medication, such as pyridoxine, to control nausea and vomiting d. Enteral nutrition to correct nutritional deficits

B Initially, the woman who is unable to down clear liquids by mouth requires IV therapy to correct fluid and electrolyte imbalances. Corticosteroids have been successfully used to treat refractory hyperemesis gravidarum, but they are not the expected initial treatment for this disorder. Pyridoxine is vitamin B6, not an antiemetic medication. Promethazine, a common antiemetic, may be prescribed. In severe cases of hyperemesis gravidarum, enteral nutrition via a feeding tube may be necessary to correct maternal nutritional deprivation but is not the initial treatment for this client.

14. The use of methamphetamine (meth) has been described as a significant drug problem in the United States. The nurse who provides care to this client population should be cognizant of what regarding methamphetamine use? a. Methamphetamines are similar to opiates. b. Methamphetamines are stimulants with vasoconstrictive characteristics. c. Methamphetamines should not be discontinued during pregnancy. d. Methamphetamines are associated with a low rate of relapse.

B Methamphetamines are stimulants with vasoconstrictive characteristics similar to cocaine and are similarly used. As is the case with cocaine users, methamphetamine users are urged to immediately stop all use during pregnancy. Unfortunately, because methamphetamine users are extremely psychologically addicted, the rate of relapse is extremely high.

A new mother with a thyroid disorder has come for a lactation follow-up appointment. Which thyroid disorder is a contraindication for breastfeeding? a. Hyperthyroidism b. PKU c. Hypothyroidism d. Thyroid storm

B PKU is a cause of mental retardation in infants; mothers with PKU pass on phenylalanine and therefore should elect not to breastfeed. A woman with either hyperthyroidism or hypothyroidism would have no particular reason not to breastfeed. A thyroid storm is a complication of hyperthyroidism and is not a contraindication to breastfeeding.

Preconception counseling is critical in the safe management of diabetic pregnancies. Which complication is commonly associated with poor glycemic control before and during early pregnancy? a. Frequent episodes of maternal hypoglycemia b. Congenital anomalies in the fetus c. Hydramnios d. Hyperemesis gravidarum

B Preconception counseling is particularly important since strict metabolic control before conception and in the early weeks of gestation is instrumental in decreasing the risk of congenital anomalies. Frequent episodes of maternal hypoglycemia may occur during the first trimester (not before conception) as a result of hormonal changes and the effects on insulin production and use. Hydramnios occurs approximately 10 times more often in diabetic pregnancies than in nondiabetic pregnancies. Typically, it is observed in the third trimester of pregnancy. Hyperemesis gravidarum may exacerbate hypoglycemic events because the decreased food intake by the mother and glucose transfer to the fetus contribute to hypoglycemia.

A 26-year-old primigravida has come to the clinic for her regular prenatal visit at 12 weeks. She appears thin and somewhat nervous. She reports that she eats a well-balanced diet, although her weight is 5 pounds less than it was at her last visit. The results of laboratory studies confirm that she has a hyperthyroid condition. Based on the available data, the nurse formulates a plan of care. Which nursing diagnosis is most appropriate for the client at this time? a. Deficient fluid volume b. Imbalanced nutrition: less than body requirements c. Imbalanced nutrition: more than body requirements d. Disturbed sleep pattern

B This clients clinical cues include weight loss, which supports a nursing diagnosis of Imbalanced nutrition: less than body requirements. No clinical signs or symptoms support a nursing diagnosis of deficient fluid volume. This client reports weight loss, not weight gain. Although the client reports nervousness, the most appropriate nursing diagnosis, based on the clients other clinical symptoms, is Imbalanced nutrition: less than body requirements.

In terms of the incidence and classification of diabetes, which information should the nurse keep in mind when evaluating clients during their ongoing prenatal appointments? a. Type 1 diabetes is most common. b. Type 2 diabetes often goes undiagnosed. c. GDM means that the woman will receive insulin treatment until 6 weeks after birth. d. Type 1 diabetes may become type 2 during pregnancy.

B Type 2 diabetes often goes undiagnosed because hyperglycemia gradually develops and is often not severe. Type 2, sometimes called adult-onset diabetes, is the most common type of diabetes. GDM refers to any degree of glucose intolerance first recognized during pregnancy; insulin may or may not be needed. People do not go back and forth between type 1 and type 2 diabetes.

Which of the following statements about the prevalence of perinatal mental health problems is true? (Select all that apply.) a. The World Health Organization (WHO) recognizes postpartum depression as the leading cause of disability in women. b. Up to a ¼ of pregnant women will experience some aspect of depression during their pregnancies. c. Income status plays a significant role in the presentation of perinatal mental health problems. d. A higher incidence of postpartum depression (PPD) is found in about 50% of the population. e. Between 30 and 50 billion dollars accounts for productivity and direct medical costs related to depression in women.

BCE

A nurse is advising a pregnant patient who has a substance abuse problem about a contingency management program. Which statement identifies an aspect of this type of program? a. Pregnant woman are confined to an inpatient treatment method during their pregnancy. b. Pregnant woman are given biofeedback modalities as stimulus responses to control their addiction. c. Pregnant woman are given motivational incentives as a primary approach to stop their drug abuse problem. d. Pregnant woman must follow a strict medication nutritional program during the course of pregnancy.

C

Nurses must be cognizant of the growing problem of methamphetamine use during pregnancy. When caring for a woman who uses methamphetamines, it is important for the nurse to be aware of which factor related to the abuse of this substance? a. Methamphetamine is a depressant. b. All methamphetamines are vasodilators. c. Methamphetamine users are extremely psychologically addicted. d. Rehabilitation is usually successful.

C

Where do most deliveries for pregnant women who have mental health issues take place? a. Mental health hospital setting b. Midwife assisted births c. Community hospital settings d. Psychiatric hospitals on locked units

C

Which preexisting factor is known to increase the risk of GDM? a. Underweight before pregnancy b. Maternal age younger than 25 years c. Previous birth of large infant d. Previous diagnosis of type 2 diabetes mellitus

C A previous birth of a large infant suggests GDM. Obesity (body mass index [BMI] of 30 or greater) creates a higher risk for gestational diabetes. A woman younger than 25 years is not generally at risk for GDM. The person with type 2 diabetes mellitus already has diabetes and thus will continue to have it after pregnancy. Insulin may be required during pregnancy because oral hypoglycemia drugs are contraindicated during pregnancy.

During a prenatal visit, the nurse is explaining dietary management to a woman with pregestational diabetes. Which statement by the client reassures the nurse that teaching has been effective? a. I will need to eat 600 more calories per day because I am pregnant. b. I can continue with the same diet as before pregnancy as long as it is well balanced. c. Diet and insulin needs change during pregnancy. d. I will plan my diet based on the results of urine glucose testing.

C Diet and insulin needs change during the pregnancy in direct correlation to hormonal changes and energy needs. In the third trimester, insulin needs may double or even quadruple. The diet is individualized to allow for increased fetal and metabolic requirements, with consideration of such factors as prepregnancy weight and dietary habits, overall health, ethnic background, lifestyle, stage of pregnancy, knowledge of nutrition, and insulin therapy. Energy needs are usually calculated on the basis of 30 to 35 calories per kilogram of ideal body weight. Dietary management during a diabetic pregnancy must be based on blood, not urine, glucose changes.

12. A pregnant woman who abuses cocaine admits to exchanging sex to finance her drug habit. This behavior places the client at the greatest risk for what? a. Depression of the CNS b. Hypotension and vasodilation c. Sexually transmitted infections (STIs) d. Postmature birth

C Exchanging sex acts for drugs places the woman at increased risk for STIs because of multiple partners and the lack of protection. Cocaine is a CNS stimulant that causes hypertension and vasoconstriction. Premature delivery of the infant is one of the more common problems associated with cocaine use during pregnancy.

To manage her diabetes appropriately and to ensure a good fetal outcome, how would the pregnant woman with diabetes alter her diet? a. Eat six small equal meals per day. b. Reduce the carbohydrates in her diet. c. Eat her meals and snacks on a fixed schedule. d. Increase her consumption of protein.

C Having a fixed meal schedule will provide the woman and the fetus with a steady blood sugar level, provide a good balance with insulin administration, and help prevent complications. Having a fixed meal schedule is more important than the equal division of food intake. Approximately 45% of the food eaten should be in the form of carbohydrates.

Which statement regarding the laboratory test for glycosylated hemoglobin Alc is correct? a. The laboratory test for glycosylated hemoglobin Alc is performed for all pregnant women, not only those with or likely to have diabetes. b. This laboratory test is a snapshot of glucose control at the moment. c. This laboratory test measures the levels of hemoglobin Alc, which should remain at less than 7%. d. This laboratory test is performed on the womans urine, not her blood.

C Hemoglobin Alc levels greater than 7% indicate an elevated glucose level during the previous 4 to 6 weeks. This extra laboratory test is for diabetic women and defines glycemic control over the previous 4 to 6 weeks. Glycosylated hemoglobin level tests are performed on the blood.

11. A woman at 24 weeks of gestation states that she has a glass of wine with dinner every evening. Why would the nurse counsel the client to eliminate all alcohol? a. Daily consumption of alcohol indicates a risk for alcoholism. b. She will be at risk for abusing other substances as well. c. The fetus is placed at risk for altered brain growth. d. The fetus is at risk for multiple organ anomalies.

C No period exists when consuming alcohol during pregnancy is safe. The documented effects of alcohol consumption during pregnancy include mental retardation, learning disabilities, high activity level, and short attention span. The brain grows most rapidly in the third trimester and is vulnerable to alcohol exposure during this time. Abuse of other substances has not been linked to alcohol use.

A number of metabolic changes occur throughout pregnancy. Which physiologic adaptation of pregnancy will influence the nurses plan of care? a. Insulin crosses the placenta to the fetus only in the first trimester, after which the fetus secretes its own. b. Women with insulin-dependent diabetes are prone to hyperglycemia during the first trimester because they are consuming more sugar. c. During the second and third trimesters, pregnancy exerts a diabetogenic effect that ensures an abundant supply of glucose for the fetus. d. Maternal insulin requirements steadily decline during pregnancy.

C Pregnant women develop increased insulin resistance during the second and third trimesters. Insulin never crosses the placenta; the fetus starts making its own around the 10th week. As a result of normal metabolic changes during pregnancy, insulin-dependent women are prone to hypoglycemia (low levels). Maternal insulin requirements may double or quadruple by the end of pregnancy.

2. When a woman is diagnosed with postpartum depression (PPD) with psychotic features, what is the nurse's primary concern in planning the client's care? a. Displaying outbursts of anger b. Neglecting her hygiene c. Harming her infant d. Losing interest in her husband

C Thoughts of harm to herself or to the infant are among the most serious symptoms of PPD and require immediate assessment and intervention. Although outbursts of anger and neglecting personal hygiene are symptoms attributable to PPD, the major concern remains the potential of harm to herself or her infant. Although this client is likely to lose interest in her spouse, it is not the nurse's primary concern.

Hypothyroidism occurs in 2 to 3 pregnancies per 1000. Because severe hypothyroidism is associated with infertility and miscarriage, it is not often seen in pregnancy. Regardless of this fact, the nurse should be aware of the characteristic symptoms of hypothyroidism. Which do they include? (Select all that apply.) a. Hot flashes b. Weight loss c. Lethargy d. Decrease in exercise capacity e. Cold intolerance

C, D, E Symptoms include weight gain, lethargy, decrease in exercise capacity, and intolerance to cold. Other presentations might include constipation, hoarseness, hair loss, and dry skin. Thyroid supplements are used to treat hyperthyroidism in pregnancy.

To provide adequate postpartum care, the nurse should be aware that postpartum depression (PPD) with psychotic features: A. Is more likely to occur in women with more than two children. B. Is rarely delusional and then usually about someone trying to harm her (the mother). C. Although serious, is not likely to need psychiatric hospitalization. D. May include bipolar disorder (formerly called "manic depression").

D

A woman with gestational diabetes has had little or no experience reading and interpreting glucose levels. The client shows the nurse her readings for the past few days. Which reading signals the nurse that the client may require an adjustment of insulin or carbohydrates? a. 75 mg/dl before lunch. This is low; better eat now. b. 115 mg/dl 1 hour after lunch. This is a little high; maybe eat a little less next time. c. 115 mg/dl 2 hours after lunch. This is too high; it is time for insulin. d. 50 mg/dl just after waking up from a nap. This is too low; maybe eat a snack before going to sleep.

D 50 mg/dl after waking from a nap is too low. During hours of sleep, glucose levels should not be less than 60 mg/dl. Snacks before sleeping can be helpful. The premeal acceptable range is 60 to 99 mg/dl. The readings 1 hour after a meal should be less than 129 mg/dl. Two hours after eating, the readings should be less than 120 mg/dl.

10. As part of the discharge teaching, the nurse can prepare the mother for her upcoming adjustment to her new role by instructing her regarding self-care activities to help prevent PPD. Which statement regarding this condition is most helpful for the client? a. Stay home, and avoid outside activities to ensure adequate rest. b. Be certain that you are the only caregiver for your baby to facilitate infant attachment. c. Keep your feelings of sadness and adjustment to your new role to yourself. d. Realize that PPD is a common occurrence that affects many women.

D Should the new mother experience symptoms of the baby blues, it is important that she be aware that these symptoms are nothing to be ashamed of. As many as 10% to 15% of new mothers experience similar symptoms. Although obtaining enough rest is important for the mother, she should not distance herself from her family and friends. Her spouse or partner can communicate the best visiting times to enable the new mother to obtain adequate rest. It is also important that she not isolate herself at home by herself during this time of role adjustment. Even if breastfeeding, other family members can participate in the infant's care. If depression occurs, then the symptoms will often interfere with mothering functions; therefore, family support is essential. The new mother should share her feelings with someone else and avoid overcommitting herself or feel as though she has to be superwoman. A telephone call to the hospital "warm line" may provide reassurance with lactation issues and other infant care questions. Should symptoms continue, a referral to a professional therapist may be necessary.

A family is visiting two surviving triplets. The third triplet died 2 days ago. What action would indicate that the family had begun to grieve for the dead infant? a. They refer to the two live infants as twins. b. They ask about the dead triplet's current status. c. They bring in play clothes for all three infants. d. They refer to the dead infant in the past tense.

D. They refer to the dead infant in the past tense Accepting that the infant is dead (in the past tense of the word) demonstrates acceptance of the reality and that the family has begun to grieve. Referring to the two live infants as twins, asking about the dead infant's current status, and bringing clothing for all three infants indicate that the parents are still in denial.

What defines gestational hypertension as opposed to preeclampsia?

It is the onset of htn after week 20 but absent of protienuria and other diagnostics for preeclampsia > 140/90

Early postpartum hemorrhage is defined as a blood loss greater than: a. 500 mL in the first 24 hours after vaginal delivery. b. 750 mL in the first 24 hours after vaginal delivery. c. 1000 mL in the first 48 hours after cesarean delivery. d. 1500 mL in the first 48 hours after cesarean delivery.

a. 500 mL in the first 24 hours after vaginal delivery.

What PPH conditions are considered medical emergencies that require immediate treatment? a. Inversion of the uterus and hypovolemic shock b. Hypotonic uterus and coagulopathies c. Subinvolution of the uterus and idiopathic thrombocytopenic purpura d. Uterine atony and disseminated intravascular coagulation

a. Inversion of the uterus and hypovolemic shock Inversion of the uterus and hypovolemic shock are considered medical emergencies. Although hypotonic uterus and coagulopathies, subinvolution of the uterus and idiopathic thrombocytopenic purpura, and uterine atony and disseminated intravascular coagulation are serious conditions, they are not necessarily medical emergencies that requires immediate treatment.

22. Which analysis of maternal serum may predict chromosomal abnormalities in the fetus? a. Multiple-marker screening b. Lecithin/sphingomyelin (L/S) ratio c. Biophysical profile d. Type and crossmatch of maternal and fetal serum

a. Multiple-marker screening

9. A woman is undergoing a nipple-stimulated contraction stress test (CST). She is having contractions that occur every 3 minutes. The fetal heart rate (FHR) has a baseline of approximately 120 beats/min without any decelerations. The interpretation of this test is said to be: a. Negative. b. Positive. c. Satisfactory. d. Unsatisfactory.

a. Negative.

According to Beck's studies, what risk factor for postpartum depression is likely to have the greatest effect on the woman's condition? a. Prenatal depression b. Single-mother status c. Low socioeconomic status d. Unplanned or unwanted pregnancy

a. Prenatal depression Prenatal depression has been found by Beck to have the greatest likely effect. Single-mother status and low socioeconomic status are small-relation predictors, as is an unwanted pregnancy.

The perinatal nurse is caring for a woman in the immediate postbirth period. Assessment reveals that the woman is experiencing profuse bleeding. The most likely etiology for the bleeding is: a. Uterine atony. b. Uterine inversion. c. Vaginal hematoma. d. Vaginal laceration.

a. Uterine atony. Uterine atony is marked hypotonia of the uterus. It is the leading cause of postpartum hemorrhage. Uterine inversion may lead to hemorrhage, but it is not the most likely source of this client's bleeding. Furthermore, if the woman were experiencing a uterine inversion, it would be evidenced by the presence of a large, red, rounded mass protruding from the introitus. A vaginal hematoma may be associated with hemorrhage. However, the most likely clinical finding would be pain, not the presence of profuse bleeding. A vaginal laceration may cause hemorrhage, but it is more likely that profuse bleeding would result from uterine atony. A vaginal laceration should be suspected if vaginal bleeding continues in the presence of a firm, contracted uterine fundus.

A newborn in the neonatal intensive care unit (NICU) is dying as a result of a massive infection. The parents speak to the neonatologist, who informs them of their son's prognosis. When the father sees his son, he says, "He looks just fine to me. I can't understand what all this is about." The most appropriate response by the nurse would be: a. "Didn't the doctor tell you about your son's problems?" b. "This must be a difficult time for you. Tell me how you're doing." c. To stand beside him quietly. d. "You'll have to face up to the fact that he is going to die sooner or later."

b. "This must be a difficult time for you. Tell me how you're doing." The grief phase can be very difficult, especially for fathers. Parents should be encouraged to share their feelings as the initial steps in the grieving process. This father is in a phase of acute distress and is "reaching out" to the nurse as a source of direction in his grieving process. Shifting the focus is not in the best interest of the parent. Nursing actions may help the parents actualize the loss of their infant through sharing and verbalization of feelings of grief. "You'll have to face up to the fact that he is going to die sooner or later" is dispassionate and inappropriate statement.

Which woman is at greatest risk for early postpartum hemorrhage (PPH)? a. A primiparous woman (G 2 P 1 0 0 1) being prepared for an emergency cesarean birth for fetal distress b. A woman with severe preeclampsia who is receiving magnesium sulfate and whose labor is being induced c. A multiparous woman (G 3 P 2 0 0 2) with an 8-hour labor d. A primigravida in spontaneous labor with preterm twins

b. A woman with severe who is receiving magnesium sulfate and whose labor is being induced. Magnesium sulfate administration during labor poses a risk for PPH. Magnesium acts as a smooth muscle relaxant, thereby contributing to uterine relaxation and atony. Although many causes and risk factors are associated with PPH, the primiparous woman being prepared for an emergency c-section, the multiparous woman with 8-hour labor, and the primigravida in spontaneous labor do not pose risk factors or causes of early PPH.

A woman is diagnosed with having a stillborn. At first, she appears stunned by the news, cries a little, and then asks you to call her mother. The phase of bereavement the woman is experiencing is called: a. Anticipatory grief. b.Acute distress. c. Intense grief. d. Reorganization.

b. Acute distress. The immediate reaction to news of a perinatal loss or infant death encompasses a period of acute distress. Disbelief and denial can occur. However, parents also feel very sad and depressed. Intense outbursts of emotion and crying are normal. However, lack of affect, euphoria, and calmness may occur and may reflect numbness, denial, or personal ways of coping with stress. Anticipatory grief applies to the grief related to a potential loss of an infant. The parent grieves in preparation of the infant's possible death, although he or she clings to the hope that the child will survive. Intense grief occurs in the first few months after the death of the infant. This phase encompasses many different emotions, including loneliness, emptiness, yearning, guilt, anger, and fear. Reorganization occurs after a long and intense search for meaning. Parents are better able to function at work and home, experience a return of self-esteem and confidence, can cope with new challenges, and have placed the loss in perspective.

A woman delivered a 9-lb, 10-oz baby 1 hour ago. When you arrive to perform her 15-minute assessment, she tells you that she "feels all wet underneath." You discover that both pads are completely saturated and that she is lying in a 6-inch-diameter puddle of blood. What is your first action? a. Call for help. b.Assess the fundus for firmness. c.Take her blood pressure. d. Check the perineum for lacerations.

b. Assess the fundus for firmness.

The prevalence of urinary incontinence (UI) increases as women age, with more than one third of women in the United States suffering from some form of this disorder. The symptoms of mild to moderate UI can be successfully decreased by a number of strategies. Which of these should the nurse instruct the client to use first? a. Pelvic floor support devices b. Bladder training and pelvic muscle exercises c. Surgery d. Medications

b. Bladder training and pelvic muscle exercises Pelvic muscle exercises, known as Kegel exercises, along with bladder training can significantly decrease or entirely relieve stress incontinence in many women. Pelvic floor support devices, also known as pessaries, come in a variety of shapes and sizes. Pessaries may not be effective for all women and require scrupulous cleaning to prevent infection. Anterior and posterior repairs and even a hysterectomy may be performed. If surgical repair is performed, the nurse must focus her care on preventing infection and helping the woman avoid putting stress on the surgical site. Pharmacologic therapy includes serotonin-norepinephrine uptake inhibitors or vaginal estrogen therapy. These are not the first action a nurse should recommend.

A mother in late middle age who is certain she is not pregnant tells the nurse during an office visit that she has urinary problems and sensations of bearing down and of something in her vagina. The nurse would realize that the client most likely is suffering from: a. Pelvic relaxation. b. Cystoceles and/or rectoceles. c. Uterine displacement. d. Genital fistulas.

b. Cystoceles and/or rectoceles. Cystoceles are protrusions of the bladder downward into the vagina; rectoceles are herniations of the anterior rectal wall through a relaxed or ruptured vaginal fascia. Both can present as a bearing down sensation with urinary dysfunction. They occur more often in older women who have borne children.

A steady trickle of bright red blood from the vagina in the presence of a firm fundus suggests: a. Uterine atony. b. Lacerations of the genital tract. c. Perineal hematoma. d. Infection of the uterus.

b. Lacerations of the genital tract.

Which instructions should be included in the discharge teaching plan to assist the patient in recognizing early signs of complications? a. Palpate the fundus daily to ensure that it is soft. b. Notify the physician of any increase in the amount of lochia or a return to bright red bleeding. c. Report any decrease in the amount of brownish red lochia. d. The passage of clots as large as an orange can be expected.

b. Notify the physician of any increase in the amount of lochia or a return to bright red bleeding.

During a follow-up visit, if parents have progressed to the second stage or phase of grieving, the nurse should not expect to see: a. Guilt, particularly in the mother. b. Numbness or lack of response. c. Bitterness or irritability. d. Fear and anxiety, especially about getting pregnant again.

b. Numbness or lack of response The second phase of grieving encompasses a wide range of intense emotions, including guilt, anger, bitterness, fear, and anxiety. What the nurse would hope not to see is numbness or unresponsiveness, as if the parents were still in denial or shock.

A primary nursing responsibility when caring for a woman experiencing an obstetric hemorrhage associated with uterine atony is to: a. Establish venous access. b. Perform fundal massage. c. Prepare the woman for surgical intervention. d. Catheterize the bladder.

b. Perform fundal massage. The initial management of excessive postpartum bleeding is firm massage of the uterine fundus. Although establishing venous access may be a necessary intervention, the initial intervention would be fundal massage. The woman may need surgical intervention to treat her postpartum hemorrhage, but the initial nursing intervention would be to assess the uterus. After uterine massage the nurse may want to catheterize the client to eliminate any bladder distention that may be preventing the uterus from contracting properly.

The perinatal nurse assisting with establishing lactation is aware that acute mastitis can be minimized by: a. Washing the nipples and breasts with mild soap and water once a day. b. Using proper breastfeeding techniques. c. Wearing a nipple shield for the first few days of breastfeeding. d. Wearing a supportive bra 24 hours a day.

b. Using proper breastfeeding techniques. Almost all instances of acute mastitis can be avoided by proper breastfeeding technique to prevent cracked nipples. Washing the nipples and breasts daily is no longer indicated. In fact, this can cause tissue dryness and irritation, which can lead to tissue breakdown and infection. Wearing a nipple shield does not prevent mastitis. Wearing a supportive bra 24 hours a day may contribute to mastitis, especially if an underwire bra is worn, because it may put pressure on the upper, outer area of the breast, contributing to blocked ducts and mastitis.

The nurse should be aware that a pessary would be most effective in the treatment of what disorder? a. Cystocele b. Uterine prolapse c. Rectocele d. Stress urinary incontinence

b. Uterine prolapse A fitted pessary may be inserted into the vagina to support the uterus and hold it in the correct position. A pessary is not used for a cystocele, a rectocele, or stress urinary incontinence.

Which options for saying goodbye would the nurse want to discuss with a woman who is diagnosed with having a stillborn girl? a. The nurse shouldn't discuss any options at this time; there is plenty of time after the baby is born. b. "Would you like a picture taken of your baby after birth?" c. "When your baby is born, would you like to see and hold her?" d. "What funeral home do you want notified after the baby is born?"

c. "When your baby is born, would you like to see and hold her?" Mothers and fathers may find it helpful to see the infant after delivery. The parents' wishes should be respected. Interventions and support from the nursing and medical staff after a prenatal loss are extremely important in the healing of the parents. Although this may be an intervention, the initial intervention should be related directly to the parents' wishes with regard to seeing or holding their dead infant. Although this information may be relevant, it is not the most appropriate option at this time. Burial arrangements can be discussed after the infant is born.

To provide adequate postpartum care, the nurse should be aware that postpartum depression (PPD) without psychotic features: a. Means that the woman is experiencing the baby blues. In addition she has a visit with a counselor or psychologist. b. Is more common among older, Caucasian women because they have higher expectations. c. Is distinguished by irritability, severe anxiety, and panic attacks. d. Will disappear on its own without outside help.

c. Is distinguished by irritability, severe anxiety, and panic attacks. PPD is also characterized by spontaneous crying long after the usual duration of the baby blues. PPD, even without psychotic features, is more serious and persistent than postpartum baby blues. It is more common among younger mothers and African-American mothers. Most women need professional help to get through PPD, including pharmacologic intervention.

During the initial acute distress phase of grieving, parents still must make unexpected and unwanted decisions about funeral arrangements and even naming the baby. The nurse's role should be to: a. Take over as much as possible to relieve the pressure. b. Encourage grandparents to take over. c. Make sure the parents themselves approve the final decisions. d. Let them alone to work things out.

c. Make sure the parents themselves approve the final decisions. The nurse is always the client's advocate. Nurses can offer support and guidance and leave room for the same from grandparents. However, in the end nurses should strive to let the parents make the final decisions.

What infection is contracted mostly by first-time mothers who are breastfeeding? a. Endometritis b. Wound infections c. Mastitis d. Urinary tract infections

c. Mastitis Mastitis is infection in a breast, usually confined to a milk duct. Most women who suffer this are first-timers who are breastfeeding.

The first and most important nursing intervention when a nurse observes profuse postpartum bleeding is to: a. Call the woman's primary health care provider. b. Administer the standing order for an oxytocic. c. Palpate the uterus and massage it if it is boggy. d. Assess maternal blood pressure and pulse for signs of hypovolemic shock.

c. Palpate the uterus and massage it if it is boggy. The initial management of excessive postpartum bleeding is firm massage of the uterine fundus. Though calling the health care provider, administering an oxytocic, and assessing maternal BP are appropriate interventions, the primary intervention should be to assess the uterus. Uterine atony is the leading cause of postpartum hemorrhage (PPH).

Anxiety disorders are the most common mental disorders that affect women. While providing care to the maternity patient, the nurse should be aware that one of these disorders is likely to be triggered by the process of labor and birth. This disorder is: a. Phobias. b. Panic disorder. c. Post-traumatic stress disorder (PTSD). d. Obsessive-compulsive disorder (OCD).

c. Post-traumatic stress disorder (PTSD).

A mother with mastitis is concerned about breastfeeding while she has an active infection. The nurse should explain that: a. The infant is protected from infection by immunoglobulins in the breast milk. b. The infant is not susceptible to the organisms that cause mastitis c. The organisms that cause mastitis are not passed to the milk. d. The organisms will be inactivated by gastric acid.

c. The organisms that cause mastitis are not passed to the milk.

A woman who has recently given birth complains of pain and tenderness in her leg. On physical examination the nurse notices warmth and redness over an enlarged, hardened area. The nurse should suspect __________ and should confirm the diagnosis by ___________. a. Disseminated intravascular coagulation; asking for laboratory tests b. von Willebrand disease; noting whether bleeding times have been extended c. Thrombophlebitis; using real-time and color Doppler ultrasound d. Coagulopathies; drawing blood for laboratory analysis

c. Thrombophlebitis; using real-time and color Doppler ultrasound Pain and tenderness in the extremities, which show warmth, redness, and hardness, likely indicate thrombophlebitis. A Doppler ultrasound is a common noninvasive way to confirm diagnosis.

After giving birth to a stillborn infant, the woman turns to the nurse and says, "I just finished painting the baby's room. Do you think that caused my baby to die?" The nurse's best response to this woman is: a. "That's an old wives' tale; lots of women are around paint during pregnancy, and this doesn't happen to them." b. "That's not likely. Paint is associated with elevated pediatric lead levels." c. Silence. d. "I can understand your need to find an answer to what caused this. What else are you thinking about?"

d. "I can understand your need to find an answer to what caused this. What else are you thinking about?" The statement, "I can understand your need to find an answer to what caused this. What else are you thinking about?" is very appropriate for the nurse. It demonstrates caring and compassion and allows the mother to vent her thoughts and feelings, which is therapeutic in the process of grieving. The nurse should resist the temptation to give advice or to use clichés in offering support to the bereaved. In addition, trying to give bereaved parents answers when no clear answers exist or trying to squelch their guilt feeling does not help the process of grief. Trying to give bereaved parents answers when no clear answers exist does not help the grief process. In addition, this response probably would increase the mother's feelings of guilt. One of the most important goals of the nurse is to validate the experience and feelings of the parents by encouraging them to tell their stories and listening with care. The nurse should encourage the mother to express her ideas.

A woman experienced a miscarriage at 10 weeks of gestation and had a dilation and curettage (D&C). She states that she is just fine and wants to go home as soon as possible. While you are assessing her responses to her loss, she tells you that she had purchased some baby things and had picked out a name. On the basis of your assessment of her responses, what nursing intervention would you use first? a. Ready her for discharge. b. Notify pastoral care to offer her a blessing. c. Ask her whether she would like to see what was obtained from her D&C. d. Ask her what name she had picked out for her baby.

d. Ask her what name she had picked out for her baby. One way of actualizing the loss is to allow parents to name the infant. The nurse should follow this client's cues and inquire about naming the infant. The client is looking for an opportunity to express her feelings of loss. The nurse should take this opportunity to offer support by allowing the woman to talk about her feelings. Furthermore, one way of actualizing the loss is to allow parents to name the infant. The nurse should follow this client's cues and inquire about naming the infant. Although it may be therapeutic to offer religious support, the nurse should take this opportunity to offer support by allowing the woman to talk about her feelings. Furthermore, one way of actualizing the loss is to allow parents to name the infant. Asking the woman if she would like to see what was obtained from her D&C is completely inappropriate.

The most appropriate statement that the nurse can make to bereaved parents is: a. "You have an angel in heaven." b. "I understand how you must feel." c. "You're young and can have other children." d. "I'm sorry."

d. I'm sorry One of nurse's most important goals is to validate the experience and feelings of the parents by encouraging them to tell their stories and listening with care. At the very least, the nurse should acknowledge the loss with a simple but sincere comment such as, "I'm sorry." The initial impulse may be to reduce one's sense of helplessness and say or do something that you think will reduce their pain. Although such a response may seem supportive at the time, it can stifle the further expression of emotion. The nurse should resist the temptation to give advice or to use clichés when offering support to the bereaved. Saying, "You're young and can have other children" is not a therapeutic response for the nurse to make.

To provide adequate postpartum care, the nurse should be aware that postpartum depression (PPD) with psychotic features: a. Is more likely to occur in women with more than two children. b. Is rarely delusional and then is usually about someone trying to harm her (the mother). c. Although serious, is not likely to need psychiatric hospitalization. d. May include bipolar disorder (formerly called "manic depression").

d. May include bipolar disorder (formerly called "manic depression"). Manic mood swings are possible. PPD is more likely to occur in first-time mothers. Delusions may be present in 50% of women with PPD, usually about something being wrong with the infant. PPD with psychosis is a psychiatric emergency that requires hospitalization.

Which condition is a transient, self-limiting mood disorder that affects new mothers after childbirth? a. Postpartum depression b.Postpartum psychosis c. Postpartum bipolar disorder d. Postpartum blues

d. Postpartum blues

With shortened hospital stays, new mothers are often discharged before they begin to experience symptoms of the baby blues or postpartum depression. As part of the discharge teaching, the nurse can prepare the mother for this adjustment to her new role by instructing her regarding self-care activities to help prevent postpartum depression. The most accurate statement as related to these activities is to: a. Stay home and avoid outside activities to ensure adequate rest. b. Be certain that you are the only caregiver for your baby, to facilitate infant attachment. c. Keep feelings of sadness and adjustment to your new role to yourself. d. Realize that this is a common occurrence that affects many women

d. Realize that this is a common occurrence that affects many women Should the new mother experience symptoms of the baby blues, it is important that she be aware that this is nothing to be ashamed of. As many as 15% of new mothers experience similar symptoms. Although it is important for the mother to obtain enough rest, she should not distance herself from family and friends. Her spouse or partner can communicate the best visiting times so the new mother can obtain adequate rest. It is also important that she not isolate herself at home during this time of role adjustment. Even if breastfeeding, other family members can participate in the infant's care. If depression occurs, the symptoms can often interfere with mothering functions and this support will be essential. The new mother should share her feelings with someone else. It is also important that she not overcommit herself or think she has to be "superwoman." A telephone call to the hospital warm line may provide reassurance with lactation issues and other infant care questions. Should symptoms continue, a referral to a professional therapist may be necessary.

One of the first symptoms of puerperal infection to assess for in the postpartum woman is: a. Fatigue continuing for longer than 1 week. b. Pain with voiding. c. Profuse vaginal bleeding with ambulation. d. Temperature of 38° C (100.4° F) or higher on 2 successive days starting 24 hours after birth.

d. Temperature of 38° C (100.4° F) or higher on 2 successive days starting 24 hours after birth. Postpartum or puerperal infection is any clinical infection of the genital canal that occurs within 28 days after miscarriage, induced abortion, or childbirth. The definition used in the United States continues to be the presence of a fever of 38° C (100.4° F) or higher on 2 successive days of the first 10 postpartum days, starting 24 hours after birth. Fatigue would be a late finding associated with infection. Pain with voiding may indicate a urinary tract infection, but it is not typically one of the earlier symptoms of infection. Profuse lochia may be associated with endometritis, but it is not the first symptom associated with infection.

Nurses need to know the basic definitions and incidence data about postpartum hemorrhage (PPH). For instance: a. PPH is easy to recognize early; after all, the woman is bleeding. b. Traditionally it takes more than 1000 mL of blood after vaginal birth and 2500 mL after cesarean birth to define the condition as PPH. c. If anything, nurses and doctors tend to overestimate the amount of blood loss. d. Traditionally PPH has been classified as early or late with respect to birth.

d. Traditionally PPH has been classified as early or late with respect to birth. Early PPH is also known as primary, or acute, PPH; late PPH is known as secondary PPH. Unfortunately PPH can occur with little warning and often is recognized only after the mother has profound symptoms. Traditionally a 500-ml blood loss after a vaginal birth and a 1000-ml blood loss after a cesarean birth constitute PPH. Medical personnel tend to underestimate blood loss by as much as 50% in their subjective observations.

When caring for a postpartum woman experiencing hemorrhagic shock, the nurse recognizes that the most objective and least invasive assessment of adequate organ perfusion and oxygenation is: a. Absence of cyanosis in the buccal mucosa. b. Cool, dry skin c. Diminished restlessness. d. Urinary output of at least 30 mL/hr.

d. Urinary output of at least 30 mL/hr. Hemorrhage may result in hemorrhagic shock. Shock is an emergency situation in which the perfusion of body organs may become severely compromised and death may occur. The presence of adequate urinary output indicates adequate tissue perfusion. The assessment of the buccal mucosa for cyanosis can be subjective in nature. The presence of cool, pale, clammy skin would be an indicative finding associated with hemorrhagic shock. Hemorrhagic shock is associated with lethargy, not restlessness.

20. Risk factors tend to be interrelated and cumulative in their effect. While planning the care for a laboring client with diabetes mellitus, the nurse is aware that she is at a greater risk for: a. Oligohydramnios. b. Polyhydramnios. c. Postterm pregnancy. d. Chromosomal abnormalities.

b. Polyhydramnios.

13. Nurses should be aware that the biophysical profile (BPP): a. Is an accurate indicator of impending fetal well being. b. Is a compilation of health risk factors of the mother during the later stages of pregnancy. c. Consists of a Doppler blood flow analysis and an amniotic fluid index. d. Involves an invasive form of ultrasound examination.

a. Is an accurate indicator of impending fetal well being.

A woman who is at 36 weeks of gestation is having a nonstress test. Which statement indicates her correct understanding of the test? "I will need to have a full bladder for the test to be done accurately." "I should have my husband drive me home after the test because I may be nauseated." "This test will help to determine whether the baby has Down syndrome or a neural tube defect." "This test observes for fetal activity and an acceleration of the fetal heart rate to determine the well-being of the baby."

"This test observes for fetal activity and an acceleration of the fetal heart rate to determine the well-being of the baby." Rationale: The nonstress test is one of the most widely used techniques to determine fetal well-being and is accomplished by monitoring fetal heart rate in conjunction with fetal activity and movements. An ultrasound requires a full bladder. An amniocentesis is the test after which a pregnant woman should be driven home. A maternal serum alpha-fetoprotein test is used in conjunction with unconjugated estriol levels and human chorionic gonadotropin helps to detect Down syndrome.

Describe the pathophysiology of preeclampsia. What is the root cause? What is the main pathogenic factor?

*The placenta is the root cause* and, thus, the disease begins to resolve with the expelling of the placenta. Inadequate vascular remodeling in uterus = insufficient placental perfusion = endothelial cell dysfunction (release of substance toxic to endothelial cells) = This anomaly causes general vasospasm = poor tissue perfusion in all organ systems, increased peripheral resistance increased BP Increased endothelial cell permeability = loss of intravascular protein and fluid loss = less plasma volume = poor perfusion= further systemic effects Summary: *the main patho factor is not an increase in BP but poor perfusion as a result of vasospasm and reduced plasma volume*

Nursing interventions for mild gestational hypertension and preeclampsia without severe features

1. Activity restriction: complete or partial bed rest 2. maternal and fetal assessment 3. Diet: regular diet with adequate protien, calcium, folic acid, zinc, sodium, fluid intake

Goals of therapy for mild gestational hypertension and preeclampsia without severe features

1. Ensure maternal safety 2. Deliver newborn close to term

Goals of therapy for severe gestational hypertension and preeclampsia with severe features

1. Ensure maternal safety 2. formulate plan for delivery

Describe the diagnostic components/ criteria for preeclampsia

1. HTN: BP >140/90 X 2 at least 4 hrs apart 2. Protienuria: >300 mg in 24-hr specimen 3. Thrombocytopenia: platelet count < 1000,000/ uL 4. Impaired liver functions: elevated liver enzyme (transaminases) 5. Renal insufficiency: new development of serum creatinine > 1.1 mg/dL 6. Pulmonary edema 7. Cerebral/ visual disturbances

27. Transvaginal ultrasonography is often performed during the first trimester. While preparing your 6-week gestation patient for this procedure, she expresses concerns over the necessity for this test. The nurse should explain that this diagnostic test may be indicated for a number of situations (Select all that apply). a. Multifetal gestation b. Obesity c. Fetal abnormalities d. Amniotic fluid volume e. Ectopic pregnancy

A, B, C, E

19. What is the most common medical complication of pregnancy? a. Hypertension b. Hyperemesis gravidarum c. Hemorrhagic complications d. Infections

ANS: A Preeclampsia and eclampsia are two noted deadly forms of hypertension. A large percentage of pregnant women will have nausea and vomiting, but a relatively few will have the severe form called hyperemesis gravidarum. Hemorrhagic complications are the second most common medical complication of pregnancy; hypertension is the most common. Infection is a risk factor for preeclampsia. DIF: Cognitive Level: Remember REF: p. 653 TOP: Nursing Process: Assessment MSC: Client Needs: Physiologic Integrity

12. When is a prophylactic cerclage for an incompetent cervix usually placed (in weeks of gestation)? a. 12 to 14 b. 6 to 8 c. 23 to 24 d. After 24

ANS: A A prophylactic cerclage is usually placed at 12 to 14 weeks of gestation. The cerclage is electively removed when the woman reaches 37 weeks of gestation or when her labor begins. Six to 8 weeks of gestation is too early to place the cerclage. Cerclage placement is offered if the cervical length falls to less than 20 to 25 mm before 23 to 24 weeks. Although no consensus has been reached, 24 weeks is used as the upper gestational age limit for cerclage placement. DIF: Cognitive Level: Apply REF: p. 674 TOP: Nursing Process: Planning MSC: Client Needs: Health Promotion and Maintenance

18. What is the correct definition of a spontaneous termination of a pregnancy (abortion)? a. Pregnancy is less than 20 weeks. b. Fetus weighs less than 1000 g. c. Products of conception are passed intact. d. No evidence exists of intrauterine infection.

ANS: A An abortion is the termination of pregnancy before the age of viability (20 weeks). The weight of the fetus is not considered because some older fetuses may have a low birth weight. A spontaneous abortion may be complete or incomplete and may be caused by many problems, one being intrauterine infection. DIF: Cognitive Level: Remember REF: p. 669 TOP: Nursing Process: Assessment MSC: Client Needs: Health Promotion and Maintenance

8. The client is being induced in response to worsening preeclampsia. She is also receiving magnesium sulfate. It appears that her labor has not become active, despite several hours of oxytocin administration. She asks the nurse, "Why is this taking so long?" What is the nurse's most appropriate response? a. "The magnesium is relaxing your uterus and competing with the oxytocin. It may increase the duration of your labor." b. "I don't know why it is taking so long." c. "The length of labor varies for different women." d. "Your baby is just being stubborn."

ANS: A Because magnesium sulfate is a tocolytic agent, its use may increase the duration of labor. The amount of oxytocin needed to stimulate labor may be more than that needed for the woman who is not receiving magnesium sulfate. The nurse should explain to the client the effects of magnesium sulfate on the duration of labor. Although the length of labor varies for different women, the most likely reason this woman's labor is protracted is the tocolytic effects of magnesium sulfate. The behavior of the fetus has no bearing on the length of labor. DIF: Cognitive Level: Apply REF: p. 664 TOP: Nursing Process: Planning MSC: Client Needs: Health Promotion and Maintenance

Of these psychosocial factors, which has the least negative effect on the health of the mother and/or fetus? a. Moderate coffee consumption b. Moderate alcohol consumption c. Cigarette smoke d. Emotional distress

ANS: A Birth defects in humans have not been related to caffeine consumption. Pregnant women who consume more than 300 mg of caffeine daily may be at increased risk for miscarriage or intrauterine growth restriction (IUGR).

11. Which intervention is most important when planning care for a client with severe gestational hypertension? a. Induction of labor is likely, as near term as possible. b. If at home, the woman should be confined to her bed, even with mild gestational hypertension. c. Special diet low in protein and salt should be initiated. d. Vaginal birth is still an option, even in severe cases.

ANS: A By 34 weeks of gestation, the risk of continuing the pregnancy may be considered greater than the risks of a preterm birth. Strict bed rest is controversial for mild cases; some women in the hospital are even allowed to move around. Diet and fluid recommendations are essentially the same as for healthy pregnant women, although some authorities have suggested a diet high in protein. Women with severe gestational hypertension should expect a cesarean delivery. DIF: Cognitive Level: Apply REF: p. 660 TOP: Nursing Process: Planning MSC: Client Needs: Health Promotion and Maintenance

17. A woman with severe preeclampsia has been receiving magnesium sulfate by intravenous infusion for 8 hours. The nurse assesses the client and documents the following findings: temperature of 37.1° C, pulse rate of 96 beats per minute, respiratory rate of 24 breaths per minute, BP of 155/112 mm Hg, 3+ DTRs, and no ankle clonus. The nurse calls the provider with an update. The nurse should anticipate an order for which medication? a. Hydralazine b. Magnesium sulfate bolus c. Diazepam d. Calcium gluconate

ANS: A Hydralazine is an antihypertensive medication commonly used to treat hypertension in severe preeclampsia. Typically, it is administered for a systolic BP higher than 160 mm Hg or a diastolic BP higher than 110 mm Hg. An additional bolus of magnesium sulfate may be ordered for increasing signs of CNS irritability related to severe preeclampsia (e.g., clonus) or if eclampsia develops. Diazepam is sometimes used to stop or shorten eclamptic seizures. Calcium gluconate is used as the antidote for magnesium sulfate toxicity. The client is not currently displaying any signs or symptoms of magnesium toxicity. DIF: Cognitive Level: Analyze REF: p. 665 TOP: Nursing Process: Planning MSC: Client Needs: Physiologic Integrity

15. Which neonatal complications are associated with hypertension in the mother? a. Intrauterine growth restriction (IUGR) and prematurity b. Seizures and cerebral hemorrhage c. Hepatic or renal dysfunction d. Placental abruption and DIC

ANS: A Neonatal complications are related to placental insufficiency and include IUGR, prematurity, and necrotizing enterocolitis. Seizures and cerebral hemorrhage are maternal complications. Hepatic and renal dysfunction are maternal complications of hypertensive disorders in pregnancy. Placental abruption and DIC are conditions related to maternal morbidity and mortality. DIF: Cognitive Level: Understand REF: p. 667 TOP: Nursing Process: Assessment MSC: Client Needs: Physiologic Integrity

14. In caring for the woman with DIC, which order should the nurse anticipate? a. Administration of blood b. Preparation of the client for invasive hemodynamic monitoring c. Restriction of intravascular fluids d. Administration of steroids

ANS: A Primary medical management in all cases of DIC involves a correction of the underlying cause, volume replacement, blood component therapy, optimization of oxygenation and perfusion status, and continued reassessment of laboratory parameters. Central monitoring would not be initially ordered in a client with DIC because it could contribute to more areas of bleeding. Management of DIC would include volume replacement, not volume restriction. Steroids are not indicated for the management of DIC. DIF: Cognitive Level: Apply REF: pp. 685-686 TOP: Nursing Process: Planning MSC: Client Needs: Physiologic Integrity

9. What nursing diagnosis is the most appropriate for a woman experiencing severe preeclampsia? a. Risk for injury to mother and fetus, related to central nervous system (CNS) irritability b. Risk for altered gas exchange c. Risk for deficient fluid volume, related to increased sodium retention secondary to the administration of magnesium sulfate d. Risk for increased cardiac output, related to the use of antihypertensive drugs

ANS: A Risk for injury is the most appropriate nursing diagnosis for this client scenario. Gas exchange is more likely to become impaired, attributable to pulmonary edema. A risk for excess, not deficient, fluid volume, related to increased sodium retention, is increased, and a risk for decreased, not increased, cardiac output, related to the use of antihypertensive drugs, also is increased. DIF: Cognitive Level: Apply REF: p. 660 TOP: Nursing Process: Diagnosis MSC: Client Needs: Physiologic Integrity

9. With regard to hemorrhagic complications that may occur during pregnancy, what information is most accurate? a. An incompetent cervix is usually not diagnosed until the woman has lost one or two pregnancies. b. Incidences of ectopic pregnancy are declining as a result of improved diagnostic techniques. c. One ectopic pregnancy does not affect a woman's fertility or her likelihood of having a normal pregnancy the next time. d. Gestational trophoblastic neoplasia (GTN) is one of the persistently incurable gynecologic malignancies.

ANS: A Short labors and recurring losses of pregnancy at progressively earlier gestational ages are characteristics of reduced cervical competence. Because diagnostic technology is improving, more ectopic pregnancies are being diagnosed. One ectopic pregnancy places the woman at increased risk for another one. Ectopic pregnancy is a leading cause of infertility. Once invariably fatal, GTN now is the most curable gynecologic malignancy. DIF: Cognitive Level: Understand REF: p. 675 TOP: Nursing Process: Assessment MSC: Client Needs: Health Promotion and Maintenance

13. In caring for an immediate postpartum client, the nurse notes petechiae and oozing from her intravenous (IV) site. The client would be closely monitored for which clotting disorder? a. DIC b. Amniotic fluid embolism (AFE) c. Hemorrhage d. HELLP syndrome

ANS: A The diagnosis of DIC is made according to clinical findings and laboratory markers. A physical examination reveals unusual bleeding. Petechiae may appear around a blood pressure cuff on the woman's arm. Excessive bleeding may occur from the site of slight trauma such as venipuncture sites. These symptoms are not associated with AFE, nor is AFE a bleeding disorder. Hemorrhage occurs for a variety of reasons in the postpartum client. These symptoms are associated with DIC. Hemorrhage would be a finding associated with DIC and is not a clotting disorder in and of itself. HELLP syndrome is not a clotting disorder, but it may contribute to the clotting disorder DIC. DIF: Cognitive Level: Understand REF: p. 685 TOP: Nursing Process: Planning MSC: Client Needs: Physiologic Integrity

2. One of the most important components of the physical assessment of the pregnant client is the determination of BP. Consistency in measurement techniques must be maintained to ensure that the nuances in the variations of the BP readings are not the result of provider error. Which techniques are important in obtaining accurate BP readings? (Select all that apply.) a. The client should be seated. b. The client's arm should be placed at the level of the heart. c. An electronic BP device should be used. d. The cuff should cover a minimum of 60% of the upper arm. e. The same arm should be used for every reading.

ANS: A, B, E BP readings are easily affected by maternal position. Ideally, the client should be seated. An alternative position is left lateral recumbent with the arm at the level of the heart. The arm should always be held in a horizontal position at approximately the level of the heart. The same arm should be used at every visit. The manual sphygmomanometer is the most accurate device. If manual and electronic devices are used in the care setting, then the nurse must use caution when interpreting the readings. A proper size cuff should cover at least 80% of the upper arm or be approximately 1.5 times the length of the upper arm. DIF: Cognitive Level: Apply REF: p. 658 TOP: Nursing Process: Assessment MSC: Client Needs: Physiologic Integrity

MULTIPLE RESPONSE 1. Which adverse prenatal outcomes are associated with the HELLP syndrome? (Select all that apply.) a. Placental abruption b. Placenta previa c. Renal failure d. Cirrhosis e. Maternal and fetal death

ANS: A, C, E The HELLP syndrome is associated with an increased risk for adverse perinatal outcomes, including placental abruption, acute renal failure, subcapsular hepatic hematoma, hepatic rupture, recurrent preeclampsia, preterm birth, and fetal and maternal death. The HELLP syndrome is associated with an increased risk for placental abruption, not placenta previa. It is also associated with an increased risk for hepatic hematoma, not cirrhosis. DIF: Cognitive Level: Analyze REF: p. 658 TOP: Nursing Process: Assessment MSC: Client Needs: Physiologic Integrity

18. The client being cared for has severe preeclampsia and is receiving a magnesium sulfate infusion. Which new finding would give the nurse cause for concern? a. Sleepy, sedated affect b. Respiratory rate of 10 breaths per minute c. DTRs of 2 d. Absent ankle clonus

ANS: B A respiratory rate of 10 breaths per minute indicates the client is experiencing respiratory depression from magnesium toxicity. Because magnesium sulfate is a CNS depressant, the client will most likely become sedated when the infusion is initiated. DTRs of 2 and absent ankle clonus are normal findings. DIF: Cognitive Level: Understand REF: p. 664 TOP: Nursing Process: Diagnosis MSC: Client Needs: Physiologic Integrity

2. A perinatal nurse is giving discharge instructions to a woman, status postsuction, and curettage secondary to a hydatidiform mole. The woman asks why she must take oral contraceptives for the next 12 months. What is the bestresponse by the nurse? a. "If you get pregnant within 1 year, the chance of a successful pregnancy is very small. Therefore, if you desire a future pregnancy, it would be better for you to use the most reliable method of contraception available." b. "The major risk to you after a molar pregnancy is a type of cancer that can be diagnosed only by measuring the same hormone that your body produces during pregnancy. If you were to get pregnant, then it would make the diagnosis of this cancer more difficult." c. "If you can avoid a pregnancy for the next year, the chance of developing a second molar pregnancy is rare. Therefore, to improve your chance of a successful pregnancy, not getting pregnant at this time is best." d. "Oral contraceptives are the only form of birth control that will prevent a recurrence of a molar pregnancy."

ANS: B Beta-human chorionic gonadotropin (beta-hCG) hormone levels are drawn for 1 year to ensure that the mole is completely gone. The chance of developing choriocarcinoma after the development of a hydatidiform mole is increased. Therefore, the goal is to achieve a zero human chorionic gonadotropin (hCG) level. If the woman were to become pregnant, then it may obscure the presence of the potentially carcinogenic cells. Women should be instructed to use birth control for 1 year after treatment for a hydatidiform mole. The rationale for avoiding pregnancy for 1 year is to ensure that carcinogenic cells are not present. Any contraceptive method except an intrauterine device (IUD) is acceptable. DIF: Cognitive Level: Apply REF: p. 679 TOP: Nursing Process: Planning | Nursing Process: Implementation MSC: Client Needs: Physiologic Integrity

11. Which laboratory marker is indicative of DIC? a. Bleeding time of 10 minutes b. Presence of fibrin split products c. Thrombocytopenia d. Hypofibrinogenemia

ANS: B Degradation of fibrin leads to the accumulation of multiple fibrin clots throughout the body's vasculature. Bleeding time in DIC is normal. Low platelets may occur but are not indicative of DIC because they may be the result from other coagulopathies. Hypofibrinogenemia occurs with DIC. DIF: Cognitive Level: Remember REF: p. 684 TOP: Nursing Process: Assessment MSC: Client Needs: Physiologic Integrity

17. Which maternal condition always necessitates delivery by cesarean birth? a. Marginal placenta previa b. Complete placenta previa c. Ectopic pregnancy d. Eclampsia

ANS: B In complete placenta previa, the placenta completely covers the cervical os. A cesarean birth is the acceptable method of delivery. The risk of fetal death occurring is due to preterm birth. If the previa is marginal (i.e., 2 cm or greater away from the cervical os), then labor can be attempted. A cesarean birth is not indicated for an ectopic pregnancy. Labor can be safely induced if the eclampsia is under control. DIF: Cognitive Level: Understand REF: p. 681 TOP: Nursing Process: Assessment MSC: Client Needs: Physiologic Integrity

1. A pregnant woman is being discharged from the hospital after the placement of a cervical cerclage because of a history of recurrent pregnancy loss, secondary to an incompetent cervix. Which information regarding postprocedural care should the nurse emphasize in the discharge teaching? a. Any vaginal discharge should be immediately reported to her health care provider. b. The presence of any contractions, rupture of membranes (ROM), or severe perineal pressure should be reported. c. The client will need to make arrangements for care at home, because her activity level will be restricted. d. The client will be scheduled for a cesarean birth.

ANS: B Nursing care should stress the importance of monitoring for the signs and symptoms of preterm labor. Vaginal bleeding needs to be reported to her primary health care provider. Bed rest is an element of care. However, the woman may stand for periods of up to 90 minutes, which allows her the freedom to see her physician. Home uterine activity monitoring may be used to limit the woman's need for visits and to monitor her status safely at home. The cerclage can be removed at 37 weeks of gestation (to prepare for a vaginal birth), or a cesarean birth can be planned. DIF: Cognitive Level: Apply REF: p. 675 TOP: Nursing Process: Planning | Nursing Process: Implementation MSC: Client Needs: Health Promotion and Maintenance

16. In contrast to placenta previa, what is the most prevalent clinical manifestation of abruptio placentae? a. Bleeding b. Intense abdominal pain c. Uterine activity d. Cramping

ANS: B Pain is absent with placenta previa and may be agonizing with abruptio placentae. Bleeding may be present in varying degrees for both placental conditions. Uterine activity and cramping may be present with both placental conditions. DIF: Cognitive Level: Understand REF: p. 683 TOP: Nursing Process: Diagnosis MSC: Client Needs: Physiologic Integrity

In the United States today: a. More than 20% of pregnancies meet the definition of high risk to either the mother or the infant b. Other than biophysical criteria, sociodemographic factors place both the mother and baby at risk c. High risk pregnancy status extends from first confirmation of pregnancy to birth d. High risk pregnancy is less critical a medical concern because of the reduction in family size and the decrease in unwanted pregnancies

ANS: B Sociodemographic risks include lack of prenatal care, low income, marital status, and ethnicity. Approximately 500,000 of the 4 million births (12.5%) in the United States are categorized as high risk. The high risk status for the mother extends through 30 days after childbirth. The reduction in family size and the decrease in unwanted pregnancies, along with technologic advances that facilitate pregnancies in previously infertile couples and advance the potential of care, have enhanced emphasis on delivering babies safely.

4. A 26-year-old pregnant woman, gravida 2, para 1-0-0-1, is 28 weeks pregnant when she experiences bright red, painless vaginal bleeding. On her arrival at the hospital, which diagnostic procedure will the client most likely have performed? a. Amniocentesis for fetal lung maturity b. Transvaginal ultrasound for placental location c. Contraction stress test (CST) d. Internal fetal monitoring

ANS: B The presence of painless bleeding should always alert the health care team to the possibility of placenta previa, which can be confirmed through ultrasonography. Amniocentesis is not performed on a woman who is experiencing bleeding. In the event of an imminent delivery, the fetus is presumed to have immature lungs at this gestational age, and the mother is given corticosteroids to aid in fetal lung maturity. A CST is not performed at a preterm gestational age. Furthermore, bleeding is a contraindication to a CST. Internal fetal monitoring is also contraindicated in the presence of bleeding. DIF: Cognitive Level: Apply REF: p. 680 TOP: Nursing Process: Assessment MSC: Client Needs: Health Promotion and Maintenance

5. A laboring woman with no known risk factors suddenly experiences spontaneous ROM. The fluid consists of bright red blood. Her contractions are consistent with her current stage of labor. No change in uterine resting tone has occurred. The fetal heart rate (FHR) begins to decline rapidly after the ROM. The nurse should suspect the possibility of what condition? a. Placenta previa b. Vasa previa c. Severe abruptio placentae d. Disseminated intravascular coagulation (DIC)

ANS: B Vasa previa is the result of a velamentous insertion of the umbilical cord. The umbilical vessels are not surrounded by Wharton jelly and have no supportive tissue. The umbilical blood vessels thus are at risk for laceration at any time, but laceration occurs most frequently during ROM. The sudden appearance of bright red blood at the time of ROM and a sudden change in the FHR without other known risk factors should immediately alert the nurse to the possibility of vasa previa. The presence of placenta previa most likely would be ascertained before labor and is considered a risk factor for this pregnancy. In addition, if the woman had a placenta previa, it is unlikely that she would be allowed to pursue labor and a vaginal birth. With the presence of severe abruptio placentae, the uterine tonicity typically is tetanus (i.e., a boardlike uterus). DIC is a pathologic form of diffuse clotting that consumes large amounts of clotting factors, causing widespread external bleeding, internal bleeding, or both. DIC is always a secondary diagnosis, often associated with obstetric risk factors such as the hemolysis, elevated liver enzyme levels, and low platelet levels (HELLP) syndrome. This woman did not have any prior risk factors. DIF: Cognitive Level: Analyze REF: p. 684 TOP: Nursing Process: Diagnosis MSC: Client Needs: Physiologic Integrity

15. A woman arrives at the emergency department with complaints of bleeding and cramping. The initial nursing history is significant for a last menstrual period 6 weeks ago. On sterile speculum examination, the primary care provider finds that the cervix is closed. The anticipated plan of care for this woman would be based on a probable diagnosis of which type of spontaneous abortion? a. Incomplete b. Inevitable c. Threatened d. Septic

ANS: C A woman with a threatened abortion has spotting, mild cramps, and no cervical dilation. A woman with an incomplete abortion would have heavy bleeding, mild-to-severe cramping, and cervical dilation. An inevitable abortion demonstrates the same symptoms as an incomplete abortion: heavy bleeding, mild-to-severe cramping, and cervical dilation. A woman with a septic abortion has malodorous bleeding and typically a dilated cervix. DIF: Cognitive Level: Understand REF: p. 670 TOP: Nursing Process: Planning MSC: Client Needs: Physiologic Integrity

6. A woman with worsening preeclampsia is admitted to the hospital's labor and birth unit. The physician explains the plan of care for severe preeclampsia, including the induction of labor, to the woman and her husband. Which statement by the husband leads the nurse to believe that the couple needs further information? a. "I will help my wife use the breathing techniques that we learned in our childbirth classes." b. "I will give my wife ice chips to eat during labor." c. "Since we will be here for a while, I will call my mother so she can bring the two boys—2 years and 4 years of age—to visit their mother." d. "I will stay with my wife during her labor, just as we planned."

ANS: C Arranging a visit with their two children indicates that the husband does not understand the importance of the quiet, subdued environment that is needed to prevent his wife's condition from worsening. Implementing breathing techniques is indicative of adequate knowledge related to pain management during labor. Administering ice chips indicates an understanding of nutritional needs during labor. Staying with his wife during labor demonstrates the husband's support for his wife and is appropriate. DIF: Cognitive Level: Apply REF: p. 662 TOP: Nursing Process: Evaluation MSC: Client Needs: Psychosocial Integrity

6. A woman arrives for evaluation of signs and symptoms that include a missed period, adnexal fullness, tenderness, and dark red vaginal bleeding. On examination, the nurse notices an ecchymotic blueness around the woman's umbilicus. What does this finding indicate? a. Normal integumentary changes associated with pregnancy b. Turner sign associated with appendicitis c. Cullen sign associated with a ruptured ectopic pregnancy d. Chadwick sign associated with early pregnancy

ANS: C Cullen sign, the blue ecchymosis observed in the umbilical area, indicates hematoperitoneum associated with an undiagnosed ruptured intraabdominal ectopic pregnancy. Linea nigra on the abdomen is the normal integumentary change associated with pregnancy and exhibits a brown pigmented, vertical line on the lower abdomen. Turner sign is ecchymosis in the flank area, often associated with pancreatitis. A Chadwick sign is a blue-purple cervix that may be seen during or around the eighth week of pregnancy. DIF: Cognitive Level: Analyze REF: p. 676 TOP: Nursing Process: Assessment MSC: Client Needs: Physiologic Integrity

2. The labor of a pregnant woman with preeclampsia is going to be induced. Before initiating the oxytocin (Pitocin) infusion, the nurse reviews the woman's latest laboratory test findings, which reveal a platelet count of 90,000 mm3, an elevated aspartate aminotransaminase (AST) level, and a falling hematocrit. The laboratory results are indicative of which condition? a. Eclampsia b. Disseminated intravascular coagulation (DIC) syndrome c. Hemolysis, elevated liver enzyme levels, and low platelet levels (HELLP) syndrome d. Idiopathic thrombocytopenia

ANS: C HELLP syndrome is a laboratory diagnosis for a variant of severe preeclampsia that involves hepatic dysfunction characterized by hemolysis (H), elevated liver (EL) enzymes, and low platelets (LP). Eclampsia is determined by the presence of seizures. DIC is a potential complication associated with HELLP syndrome. Idiopathic thrombocytopenia is the presence of low platelets of unknown cause and is not associated with preeclampsia. DIF: Cognitive Level: Understand REF: p. 657 TOP: Nursing Process: Diagnosis MSC: Client Needs: Physiologic Integrity

12. What is the primary purpose for magnesium sulfate administration for clients with preeclampsia and eclampsia? a. To improve patellar reflexes and increase respiratory efficiency b. To shorten the duration of labor c. To prevent convulsions d. To prevent a boggy uterus and lessen lochial flow

ANS: C Magnesium sulfate is the drug of choice used to prevent convulsions, although it can generate other problems. Loss of patellar reflexes and respiratory depression are signs of magnesium toxicity. Magnesium sulfate can also increase the duration of labor. Women are at risk for a boggy uterus and heavy lochial flow as a result of magnesium sulfate therapy. DIF: Cognitive Level: Understand REF: p. 664 TOP: Nursing Process: Implementation MSC: Client Needs: Physiologic Integrity

3. The nurse is preparing to administer methotrexate to the client. This hazardous drug is most often used for which obstetric complication? a. Complete hydatidiform mole b. Missed abortion c. Unruptured ectopic pregnancy d. Abruptio placentae

ANS: C Methotrexate is an effective nonsurgical treatment option for a hemodynamically stable woman whose ectopic pregnancy is unruptured and measures less than 4 cm in diameter. Methotrexate is not indicated or recommended as a treatment option for a complete hydatidiform mole, for a missed abortion, or for abruptio placentae. DIF: Cognitive Level: Apply REF: p. 677 TOP: Nursing Process: Planning MSC: Client Needs: Physiologic Integrity

14. Women with mild gestational hypertension and mild preeclampsia can be safely managed at home with frequent maternal and fetal evaluation. Complete or partial bed rest is still frequently ordered by some providers. Which complication is rarely the result of prolonged bed rest? a. Thrombophlebitis b. Psychologic stress c. Fluid retention d. Cardiovascular deconditioning

ANS: C No evidence has been found that supports the practice of bed rest to improve pregnancy outcome. Fluid retention is not an adverse outcome of prolonged bed rest. The woman is more likely to experience diuresis with accompanying fluid and electrolyte imbalance and weight loss. Prolonged bed rest is known to increase the risk for thrombophlebitis. Psychologic stress is known to begin on the first day of bed rest and continue for the duration of the therapy. Therefore, restricted activity, rather than complete bed rest, is recommended. Cardiovascular deconditioning is a known complication of bed rest. DIF: Cognitive Level: Understand REF: p. 661 TOP: Nursing Process: Diagnosis MSC: Client Needs: Physiologic Integrity

10. The management of the pregnant client who has experienced a pregnancy loss depends on the type of miscarriage and the signs and symptoms. While planning care for a client who desires outpatient management after a first-trimester loss, what would the nurse expect the plan to include? a. Dilation and curettage (D&C) b. Dilation and evacuation (D&E) c. Misoprostol d. Ergot products

ANS: C Outpatient management of a first-trimester loss is safely accomplished by the intravaginal use of misoprostol for up to 2 days. If the bleeding is uncontrollable, vital signs are unstable, or signs of infection are present, then a surgical evacuation should be performed. D&C is a surgical procedure that requires dilation of the cervix and scraping of the uterine walls to remove the contents of pregnancy. This procedure is commonly performed to treat inevitable or incomplete abortion and should be performed in a hospital. D&E is usually performed after 16 weeks of pregnancy. The cervix is widely dilated, followed by removal of the contents of the uterus. Ergot products such as Methergine or Hemabate may be administered for excessive bleeding after miscarriage. DIF: Cognitive Level: Apply REF: p. 672 TOP: Nursing Process: Planning MSC: Client Needs: Physiologic Integrity

1. A primigravida is being monitored at the prenatal clinic for preeclampsia. Which finding is of greatest concern to the nurse? a. Blood pressure (BP) increase to 138/86 mm Hg b. Weight gain of 0.5 kg during the past 2 weeks c. Dipstick value of 3+ for protein in her urine d. Pitting pedal edema at the end of the day

ANS: C Proteinuria is defined as a concentration of 1+ or greater via dipstick measurement. A dipstick value of 3+ alerts the nurse that additional testing or assessment should be performed. A 24-hour urine collection is preferred over dipstick testing attributable to accuracy. Generally, hypertension is defined as a BP of 140/90 mm Hg or an increase in systolic pressure of 30 mm Hg or diastolic pressure of 15 mm Hg. Preeclampsia may be demonstrated as a rapid weight gain of more than 2 kg in 1 week. Edema occurs in many normal pregnancies, as well as in women with preeclampsia. Therefore, the presence of edema is no longer considered diagnostic of preeclampsia. DIF: Cognitive Level: Analyze REF: p. 660 TOP: Nursing Process: Diagnosis MSC: Client Needs: Physiologic Integrity

4. A pregnant woman has been receiving a magnesium sulfate infusion for treatment of severe preeclampsia for 24 hours. On assessment, the nurse finds the following vital signs: temperature 37.3° C, pulse rate 88 beats per minute, respiratory rate 10 breaths per minute, BP 148/90 mm Hg, absent deep tendon reflexes (DTRs), and no ankle clonus. The client complains, "I'm so thirsty and warm." What is the nurse's immediate action? a. To call for an immediate magnesium sulfate level b. To administer oxygen c. To discontinue the magnesium sulfate infusion d. To prepare to administer hydralazine

ANS: C Regardless of the magnesium level, the client is displaying the clinical signs and symptoms of magnesium toxicity. The first action by the nurse should be to discontinue the infusion of magnesium sulfate. In addition, calcium gluconate, the antidote for magnesium, may be administered. Hydralazine is an antihypertensive drug commonly used to treat hypertension in severe preeclampsia. Typically, hydralazine is administered for a systolic BP higher than 160 mm Hg or a diastolic BP higher than 110 mm Hg. DIF: Cognitive Level: Apply REF: p. 664 TOP: Nursing Process: Implementation MSC: Client Needs: Physiologic Integrity

8. A woman who is 30 weeks of gestation arrives at the hospital with bleeding. Which differential diagnosis would not be applicable for this client? a. Placenta previa b. Abruptio placentae c. Spontaneous abortion d. Cord insertion

ANS: C Spontaneous abortion is another name for miscarriage; it occurs, by definition, early in pregnancy. Placenta previa is a well-known reason for bleeding late in pregnancy. The premature separation of the placenta (abruptio placentae) is a bleeding disorder that can occur late in pregnancy. Cord insertion may cause a bleeding disorder that can also occur late in pregnancy. DIF: Cognitive Level: Understand REF: p. 669 TOP: Nursing Process: Assessment MSC: Client Needs: Physiologic Integrity, Physiologic Adaptation

20. Which statement most accurately describes the HELLP syndrome? a. Mild form of preeclampsia b. Diagnosed by a nurse alert to its symptoms c. Characterized by hemolysis, elevated liver enzymes, and low platelets d. Associated with preterm labor but not perinatal mortality

ANS: C The acronym HELLP stands for hemolysis (H), elevated liver (EL) enzymes, and low platelets (LP). The HELLP syndrome is a variant of severe preeclampsia and is difficult to identify because the symptoms are not often obvious. The HELLP syndrome must be diagnosed in the laboratory. Preterm labor is greatly increased; therefore, so is perinatal mortality. DIF: Cognitive Level: Understand REF: p. 657 TOP: Nursing Process: Diagnosis | Nursing Process: Planning MSC: Client Needs: Physiologic Integrity

6.A 30-year-old gravida 3, para 2-0-0-2 is at 18 weeks of gestation. What screening test should be suggested to her? a. Biophysical profile b. Chorionic villi sampling c. Maternal serum alpha-fetoprotein (MSAFP) screening d. Screening for diabetes mellitus

ANS: C The biochemical assessment MSAFP test is performed from week 15 to week 20 of gestation (weeks 16 to 18 are ideal). A biophysical profile is a method of biophysical assessment of fetal well-being in the third trimester. Chorionic villi sampling is a biochemical assessment of the fetus that should be performed from the tenth to twelfth weeks of gestation. Screening for diabetes mellitus begins with the first prenatal visit.

13. The American College of Obstetricians and Gynecologists (ACOG) has developed a comprehensive list of risk factors associated with the development of preeclampsia. Which client exhibits the greatest number of these risk factors? a. 30-year-old obese Caucasian with her third pregnancy b. 41-year-old Caucasian primigravida c. 19-year-old African American who is pregnant with twins d. 25-year-old Asian American whose pregnancy is the result of donor insemination

ANS: C Three risk factors are present in the 19-year-old African-American client. She has African-American ethnicity, is at the young end of the age distribution, and has a multiple pregnancy. In planning care for this client, the nurse must frequently monitor her BP and teach her to recognize the early warning signs of preeclampsia. The 30-year-old obese Caucasian client has only has one known risk factor: obesity. Age distribution appears to be U-shaped, with women younger than 20 years of age and women older than 40 years of age being at greatest risk. Preeclampsia continues to be more frequently observed in primigravidas; this client is a multigravida woman. Two risk factors are present for the 41-year-old Caucasian primigravida client. Her age and status as a primigravida place her at increased risk for preeclampsia. Caucasian women are at a lower risk than are African-American women. The 25-year-old Asian-American client exhibits only one risk factor. Pregnancies that result from donor insemination, oocyte donation, and embryo donation are at an increased risk of developing preeclampsia. DIF: Cognitive Level: Analyze REF: p. 655 TOP: Nursing Process: Planning MSC: Client Needs: Physiologic Integrity

7. The nurse who elects to practice in the area of women's health must have a thorough understanding of miscarriage. Which statement regarding this condition is most accurate? a. A miscarriage is a natural pregnancy loss before labor begins. b. It occurs in fewer than 5% of all clinically recognized pregnancies. c. Careless maternal behavior, such as poor nutrition or excessive exercise, can be a factor in causing a miscarriage. d. If a miscarriage occurs before the 12th week of pregnancy, then it may be observed only as moderate discomfort and blood loss.

ANS: D Before the sixth week, the only evidence might be a heavy menstrual flow. After the 12th week, more severe pain, similar to that of labor, is likely. Miscarriage is a natural pregnancy loss, but it occurs, by definition, before 20 weeks of gestation, before the fetus is viable. Miscarriages occur in approximately 10% to 15% of all clinically recognized pregnancies. Miscarriages can be caused by a number of disorders or illnesses outside the mother's control or knowledge. DIF: Cognitive Level: Understand REF: p. 670 TOP: Nursing Process: Assessment MSC: Client Needs: Physiologic Integrity

7. The client has been on magnesium sulfate for 20 hours for the treatment of preeclampsia. She just delivered a viable infant girl 30 minutes ago. What uterine findings does the nurse expect to observe or assess in this client? a. Absence of uterine bleeding in the postpartum period b. Fundus firm below the level of the umbilicus c. Scant lochia flow d. Boggy uterus with heavy lochia flow

ANS: D High serum levels of magnesium can cause a relaxation of smooth muscle such as the uterus. Because of this tocolytic effect, the client will most likely have a boggy uterus with increased amounts of bleeding. All women experience uterine bleeding in the postpartum period, especially those who have received magnesium therapy. Rather than scant lochial flow, however, this client will most likely have a heavy flow attributable to the relaxation of the uterine wall caused by magnesium administration. DIF: Cognitive Level: Analyze REF: p. 664 TOP: Nursing Process: Assessment MSC: Client Needs: Physiologic Integrity

3. A woman with preeclampsia has a seizure. What is the nurse's highest priority during a seizure? a. To insert an oral airway b. To suction the mouth to prevent aspiration c. To administer oxygen by mask d. To stay with the client and call for help

ANS: D If a client becomes eclamptic, then the nurse should stay with the client and call for help. Nursing actions during a convulsion are directed toward ensuring a patent airway and client safety. Insertion of an oral airway during seizure activity is no longer the standard of care. The nurse should attempt to keep the airway patent by turning the client's head to the side to prevent aspiration. Once the seizure has ended, it may be necessary to suction the client's mouth. Oxygen is administered after the convulsion has ended. DIF: Cognitive Level: Apply REF: p. 666 TOP: Nursing Process: Implementation MSC: Client Needs: Physiologic Integrity

With regard to maternal, fetal, and neonatal health problems, nurses should be aware that: a. Infection has replaced pulmonary embolism as one of the three top causes of maternal death attributable to pregnancy b. The leading cause of death in the neonatal period is disorders related to short gestation and low birth weight c. Factors related to the maternal death rate include age and marital status but not race d. Antepartum fetal deaths can best be prevented by better recognizing and responding to abnormalities of pregnancy and labor

ANS: D Medical teams need to be alert to signs of trouble. Race is a factor. African-American maternal mortality rates are more than three times higher than those for Caucasian women. Infection used to be an important cause of maternal death; it has been replaced by pulmonary embolism. The leading cause of death in the neonatal period is congenital anomalies. Race is a factor. African-American maternal mortality rates are more than three times higher than those for Caucasian women.

5. A woman at 39 weeks of gestation with a history of preeclampsia is admitted to the labor and birth unit. She suddenly experiences increased contraction frequency of every 1 to 2 minutes, dark red vaginal bleeding, and a tense, painful abdomen. Which clinical change does the nurse anticipate? a. Eclamptic seizure b. Rupture of the uterus c. Placenta previa d. Abruptio placentae

ANS: D Uterine tenderness in the presence of increasing tone may be the earliest sign of abruptio placentae. Women with preeclampsia are at increased risk for an abruption attributable to decreased placental perfusion. Eclamptic seizures are evidenced by the presence of generalized tonic-clonic convulsions. Uterine rupture exhibits hypotonic uterine activity, signs of hypovolemia, and, in many cases, the absence of pain. Placenta previa exhibits bright red, painless vaginal bleeding. DIF: Cognitive Level: Understand REF: p. 662 TOP: Nursing Process: Diagnosis MSC: Client Needs: Physiologic Integrity

10. Which statement best describes chronic hypertension? a. Chronic hypertension is defined as hypertension that begins during pregnancy and lasts for the duration of the pregnancy. b. Chronic hypertension is considered severe when the systolic BP is higher than 140 mm Hg or the diastolic BP is higher than 90 mm Hg. c. Chronic hypertension is general hypertension plus proteinuria. d. Chronic hypertension can occur independently of or simultaneously with preeclampsia.

ANS: D Women with chronic hypertension may develop superimposed preeclampsia, which increases the morbidity for both the mother and the fetus. Chronic hypertension is present before pregnancy or diagnosed before the 20 weeks of gestation and persists longer than 6 weeks postpartum. Chronic hypertension becomes severe with a diastolic BP of 110 mm Hg or higher. Proteinuria is an excessive concentration of protein in the urine and is a complication of hypertension, not a defining characteristic. DIF: Cognitive Level: Understand REF: p. 667 TOP: Nursing Process: Diagnosis | Nursing Process: Planning MSC: Client Needs: Physiologic Integrity

Which assessment is not included in the fetal biophysical profile (BPP)? a. Fetal movement b. Fetal tone c. Fetal heart rate d. Amniotic fluid index e. Placental grade

ANS: E Fetal movement, tone, heart rate, and amniotic fluid index are all assessed in a BPP. The placental grade is determined by ultrasound (as is a BPP), but it is not included in the criteria of assessment factors for a BPP.

2. A 39-year-old primigravida thinks that she is about 8 weeks pregnant, although she has had irregular menstrual periods all her life. She has a history of smoking approximately one pack of cigarettes a day, but she tells you that she is trying to cut down. Her laboratory data are within normal limits. What diagnostic technique could be used with this pregnant woman at this time? a. Ultrasound examination b. Maternal serum alpha-fetoprotein (MSAFP) screening c. Amniocentesis d. Nonstress test (NST)

a. Ultrasound examination

When would the best timeframe be to establish gestational age based on ultrasound? At term 8 weeks Between 14 and 22 weeks 36 weeks

Between 14 and 22 weeks Rationale: Ultrasound determination of gestational age dating is best done between 14 and 22 weeks. It is less reliable after that period because of variability in fetal size. Standard sets of measurements relative to gestational age are noted around 10 to after 12 weeks and include crown-rump length (after 10), biparietal diameter (after 12), femur length, and head and abdominal circumferences.

A 40-year-old woman with a high body mass index (BMI) is 10 weeks pregnant. Which diagnostic tool is appropriate to suggest to her at this time? Biophysical profile Incorrect Amniocentesis Maternal serum alpha-fetoprotein (MSAFP) Transvaginal ultrasound

D. Transvaginal ultrasound Rationale: An ultrasound is the method of biophysical assessment of the infant that is performed at this gestational age. Transvaginal ultrasound is especially useful for obese women, whose thick abdominal layers cannot be penetrated adequately with the abdominal approach. A biophysical profile is a method of biophysical assessment of fetal well-being in the third trimester. An amniocentesis is performed after the fourteenth week of pregnancy. A MSAFP test is performed from week 15 to week 22 of the gestation (weeks 16 to 18 are ideal).

The nurse sees a woman for the first time when she is 30 weeks pregnant. The woman has smoked throughout the pregnancy, and fundal height measurements now are suggestive of growth restriction in the fetus. In addition to ultrasound to measure fetal size, what would be another tool useful in confirming the diagnosis? Doppler blood flow analysis Contraction stress test (CST) Amniocentesis Daily fetal movement counts

Doppler blood flow analysis Rationale: Doppler blood flow analysis allows the examiner to study the blood flow noninvasively in the fetus and the placenta. It is a helpful tool in the management of high risk pregnancy due to intrauterine growth restriction (IUGR), diabetes mellitus, multiple fetuses, or preterm labor. Because of the potential risk of inducing labor and causing fetal distress, a CST is not performed in a woman whose fetus is preterm. Indications for an amniocentesis include diagnosis of genetic disorders or congenital anomalies, assessment of pulmonary maturity, and the diagnosis of fetal hemolytic disease, not IUGR. Fetal kick count monitoring is performed to monitor the fetus in pregnancies complicated by conditions that may affect fetal oxygenation. Although it may be a useful tool at some point later in this woman's pregnancy, it is not used to diagnose IUGR.

In the past, factors to determine whether a woman was likely to have a high risk pregnancy were evaluated primarily from a medical point of view. A broader, more comprehensive approach to high risk pregnancy has been adopted. There are now four categories based on threats to the health of the woman and the outcome of pregnancy. Which of the options listed here is not included as a category? Biophysical Psychosocial Geographic Environmental

Geographic Rationale: The fourth category is correctly referred to as the sociodemographic risk category. The factors stem from the mother and her family. Ethnicity may be one of the risks to pregnancy; however, it is not the only factor in this category. Low income, lack of prenatal care, age, parity, and marital status are included. Biophysical is one of the broad categories used for determining risk. It includes genetic considerations, nutritional status, and medical and obstetric disorders. Psychosocial risks include smoking, caffeine, drugs, alcohol, and psychologic status. All of these adverse lifestyles can have a negative effect on the health of the mother or fetus. Environmental risks are those that can affect fertility and fetal development. They include infections, chemicals, radiation, pesticides, illicit drugs, and industrial pollutants.

A nurse providing care for the antepartum woman should understand that the contraction stress test (CST): Sometimes uses vibroacoustic stimulation. Is an invasive test; however, contractions are stimulated. Is considered to have a negative result if no late decelerations are observed with the contractions. Is more effective than nonstress test (NST) if the membranes have already been ruptured.

Is considered to have a negative result if no late decelerations are observed with the contractions Rationale: No late decelerations indicate a positive CST result. Vibroacoustic stimulation is sometimes used with NST. CST is invasive if stimulation is performed by IV oxytocin but not if by nipple stimulation. CST is contraindicated if the membranes have ruptured..

Nurses should be aware of the strengths and limitations of various biochemical assessments during pregnancy, including that: Chorionic villus sampling (CVS) is becoming more popular because it provides early diagnosis. Screening for maternal serum alpha-fetoprotein (MSAFP) levels is recommended only for women at risk for neural tube defects. Percutaneous umbilical blood sampling (PUBS) is one of the quad-screen tests for Down syndrome. MSAFP is a screening tool only; it identifies candidates for more definitive procedures.

MSAFP is a screening tool only; it identifies candidates for more definitive procedures. Rationale: CVS does provide a rapid result, but it is declining in popularity because of advances in noninvasive screening techniques. MSAFP screening is recommended for all pregnant women. MSAFP, not PUBS, is part of the quad-screen tests for Down syndrome. MSAFP is a screening tool, not a diagnostic tool. Further diagnostic testing is indicated after an abnormal result.

What is an appropriate indicator for performing a contraction stress test? Increased fetal movement and small for gestational age Maternal diabetes mellitus and postmaturity Adolescent pregnancy and poor prenatal care History of preterm labor and intrauterine growth restriction

Maternal diabetes mellitus and postmaturity Rationale: Decreased fetal movement is an indicator for performing a contraction stress test; the size (small for gestational age) is not an indicator. Although adolescent pregnancy and poor prenatal care are risk factors for poor fetal outcomes, they are not indicators for performing a contraction stress test. Intrauterine growth restriction is an indicator; history of a previous stillbirth, not preterm labor, is another indicator.

A patient has undergone an amniocentesis for evaluation of fetal well-being. Which intervention would be included in the nurse's plan of care after the procedure? (Select all that apply.) Perform ultrasound to determine fetal positioning. Observe the patient for possible uterine contractions. Administer RhoGAM to the patient if she is Rh negative. Perform a minicatheterization to obtain a urine specimen to assess for bleeding.

Observe the patient for possible uterine contractions. Administer RhoGAM to the patient if she is Rh negative. Rationale: Ultrasound is used prior to the procedure as a visualization aid to assist with insertion of transabdominal needle. There is no need to assess the urine for bleeding as this is not considered to be a typical presentation or complication.

What is preecampsia?

Pregnancy-specific condition in which hypertension and protienuria develop after week 20 in a normotensive pt

How does HELP syndrome occur? What are the pathophysiologic changes?

arteriolar vasospasm, endothelial cell dysfxn with fibrin deposits, and adherence of platelets in blood vessels RBC's damaged as they pass through narrowed blood vessels and become hemolyzed= decreased RBC count and hyperbilirubinemia Endothelial damage and fibrin deposits in liver= impaired liver dysfxn and can lead to hemorrhagic necrosis

4. A 41-week pregnant multigravida presents in the labor and delivery unit after a nonstress test indicated that her fetus could be experiencing some difficulties in utero. Which diagnostic tool would yield more detailed information about the fetus? a. Ultrasound for fetal anomalies b. Biophysical profile (BPP) c. Maternal serum alpha-fetoprotein (MSAFP) screening d. Percutaneous umbilical blood sampling (PUBS)

b. Biophysical profile (BPP)

18. A woman has been diagnosed with a high risk pregnancy. She and her husband come into the office in a very anxious state. She seems to be coping by withdrawing from the discussion, showing declining interest. The nurse can best help the couple by: a. Telling her that the physician will isolate the problem with more tests. b. Encouraging her and urging her to continue with childbirth classes. c. Becoming assertive and laying out the decisions the couple needs to make. d. Downplaying her risks by citing success rate studies.

b. Encouraging her and urging her to continue with childbirth classes.

A nurse is providing instruction for an obstetrical patient to perform a daily fetal movement count (DFMC). Which instructions could be included in the plan of care? (Select all that apply.) The fetal alarm signal is reached when there are no fetal movements noted for 5 hours. The patient can monitor fetal activity once daily for a 60-minute period and note activity. Monitor fetal activity two times a day either after meals or before bed for a period of 2 hours or until 10 fetal movements are noted. Count all fetal movements in a 12-hour period daily until 10 fetal movements are noted.

The patient can monitor fetal activity once daily for a 60-minute period and note activity. Monitor fetal activity two times a day either after meals or before bed for a period of 2 hours or until 10 fetal movements are noted. Count all fetal movements in a 12-hour period daily until 10 fetal movements are noted. Rationale: The fetal alarm signal is reached when no fetal movements are noted for a period of 12 hours.

What is HELP syndrome?

a laboratory diagnosis for a variant of severe preeclampsia that involves hepatic dysfxn. It is not a separate illness

21. A pregnant woman's biophysical profile score is 8. She asks the nurse to explain the results. The nurse's best response is: a. "The test results are within normal limits." b. "Immediate delivery by cesarean birth is being considered." c. "Further testing will be performed to determine the meaning of this score." d. "An obstetric specialist will evaluate the results of this profile and, within the next week, will inform you of your options regarding delivery."

a. "The test results are within normal limits."

8. A client asks her nurse, "My doctor told me that he is concerned with the grade of my placenta because I am overdue. What does that mean?" The best response by the nurse is: a. "Your placenta changes as your pregnancy progresses, and it is given a score that indicates the amount of calcium deposits it has. The more calcium deposits, the higher the grade, or number, that is assigned to the placenta. It also means that less blood and oxygen can be delivered to your baby." b. "Your placenta isn't working properly, and your baby is in danger." c. "This means that we will need to perform an amniocentesis to detect if you have any placental damage." d. "Don't worry about it. Everything is fine."

a. "Your placenta changes as your pregnancy progresses, and it is given a score that indicates the amount of calcium deposits it has. The more calcium deposits, the higher the grade, or number, that is assigned to the placenta. It also means that less blood and oxygen can be delivered to your baby."

3. The nurse sees a woman for the first time when she is 30 weeks pregnant. The woman has smoked throughout the pregnancy, and fundal height measurements now are suggestive of growth restriction in the fetus. In addition to ultrasound to measure fetal size, what other tool would be useful in confirming the diagnosis? a. Doppler blood flow analysis b. Contraction stress test (CST) c. Amniocentesis d. Daily fetal movement counts

a. Doppler blood flow analysis

16. Compared with contraction stress test (CST), nonstress test (NST) for antepartum fetal assessment: a. Has no known contraindications. b. Has fewer false-positive results. c. Is more sensitive in detecting fetal compromise. d. Is slightly more expensive.

a. Has no known contraindications.

24. Which nursing intervention is necessary before a second-trimester transabdominal ultrasound? a. Place the woman NPO for 12 hours. b. Instruct the woman to drink 1 to 2 quarts of water. c. Administer an enema. d. Perform an abdominal preparation.

b. Instruct the woman to drink 1 to 2 quarts of water.

7. Maternal serum alpha-fetoprotein (MSAFP) screening indicates an elevated level. MSAFP screening is repeated and again is reported as higher than normal. What would be the next step in the assessment sequence to determine the well-being of the fetus? a. Percutaneous umbilical blood sampling (PUBS) b. Ultrasound for fetal anomalies c. Biophysical profile (BPP) for fetal well-being d. Amniocentesis for genetic anomalies

b. Ultrasound for fetal anomalies

12. In the first trimester, ultrasonography can be used to gain information on: a. Amniotic fluid volume. b. the presence of maternal abnormalities c. Placental location and maturity. d. Cervical length.

b. the presence of maternal abnormalities

23. While working with the pregnant woman in her first trimester, the nurse is aware that chorionic villus sampling (CVS) can be performed during pregnancy at: a. 4 weeks b. 8 weeks c. 10 weeks d. 14 weeks

c. 10 weeks

14. With regard to amniocentesis, nurses should be aware that: a. Because of new imaging techniques, amniocentesis is now possible in the first trimester. b. Despite the use of ultrasound, complications still occur in the mother or infant in 5% to 10% of cases. c. Administration RhoD immunoglobulin may be necessary. d. The presence of meconium in the amniotic fluid is always cause for concern.

c. Administration RhoD immunoglobulin may be necessary.

5. At 35 weeks of pregnancy a woman experiences preterm labor. Tocolytics are administered and she is placed on bed rest, but she continues to experience regular uterine contractions, and her cervix is beginning to dilate and efface. What would be an important test for fetal well-being at this time? a. Percutaneous umbilical blood sampling (PUBS) b. Ultrasound for fetal size c. Amniocentesis for fetal lung maturity d. Nonstress test (NST)

c. Amniocentesis for fetal lung maturity

17. The nurse providing care for the antepartum woman should understand that contraction stress test (CST): a. Sometimes uses vibroacoustic stimulation. b. Is an invasive test; however, contractions are stimulated. c. Is considered negative if no late decelerations are observed with the contractions. d. Is more effective than nonstress test (NST) if the membranes have already been ruptured.

c. Is considered negative if no late decelerations are observed with the contractions.

10. When nurses help their expectant mothers assess the daily fetal movement counts, they should be aware that: a. Alcohol or cigarette smoke can irritate the fetus into greater activity. b. "Kick counts" should be taken every half hour and averaged every 6 hours, with every other 6-hour stretch off. c. The fetal alarm signal should go off when fetal movements stop entirely for 12 hours. d. Obese mothers familiar with their bodies can assess fetal movement as well as average-size women.

c. The fetal alarm signal should go off when fetal movements stop entirely for 12 hours.

1. A woman arrives at the clinic seeking confirmation that she is pregnant. The following information is obtained: She is 24 years old with a body mass index (BMI) of 17.5. She admits to having used cocaine "several times" during the past year and drinks alcohol occasionally. Her blood pressure (BP) is 108/70 mm Hg, her pulse rate is 72 beats/min, and her respiratory rate is 16 breaths/min. The family history is positive for diabetes mellitus and cancer. Her sister recently gave birth to an infant with a neural tube defect (NTD). Which characteristics place the woman in a high risk category? a. Blood pressure, age, BMI b. Drug/alcohol use, age, family history c. Family history, blood pressure, BMI d. Family history, BMI, drug/alcohol abuse

d. Family history, BMI, drug/alcohol abuse

15. Nurses should be aware of the strengths and limitations of various biochemical assessments during pregnancy, including that: a. Chorionic villus sampling (CVS) is becoming more popular because it provides early diagnosis. b. Maternal serum alpha-fetoprotein (MSAFP) screening is recommended only for women at risk for neural tube defects. c. Percutaneous umbilical blood sampling (PUBS) is one of the triple-marker tests for Down syndrome. d. MSAFP is a screening tool only; it identifies candidates for more definitive procedures.

d. MSAFP is a screening tool only; it identifies candidates for more definitive procedures.

25. The nurse recognizes that a nonstress test (NST) in which two or more fetal heart rate (FHR) accelerations of 15 beats/min or more occur with fetal movement in a 20-minute period is: a. Nonreactive b. Positive c. Negative d. Reactive

d. Reactive

11. In comparing the abdominal and transvaginal methods of ultrasound examination, nurses should explain to their clients that: a. Both require the woman to have a full bladder. b. The abdominal examination is more useful in the first trimester. c. Initially the transvaginal examination can be painful. d. The transvaginal examination allows pelvic anatomy to be evaluated in greater detail.

d. The transvaginal examination allows pelvic anatomy to be evaluated in greater detail.

What non-specific clinical presentation might a pt report as a result of HELP syndrome?

hx of malaise, flu-like symptoms, epigastric/ RUQ pain, symptoms worse at night


Conjuntos de estudio relacionados

Accounting - Financial Accounting: Information for Decisions - Wild 9e Chapter 2. Financial Statements and the Accounting System

View Set

Campbell Figure Walkthrough: The lac Operon in E. coli -- Regulated Synthesis of Inducible Enzymes (HW 16 DNA gene regulation)

View Set

Percent of Change, Percent Application

View Set

Quiz 3 dividend discount models to value stocks

View Set

the Fronde in Bordeaux - the Ormee

View Set

Topic 2 Lesson 6 New Ways of Life

View Set